Тест по физике динамика 10 класс: Тест по физике на тему «Динамика» (10 класс)

Содержание

Тест по физике на тему «Динамика» (10 класс)

Тест по теме «Динамика». Для 10 класса.

1. Инерциальная система отсчета – это система отсчета, в которой …

    А) Любое ускорение, приобретаемое телом, объясняется действием на него других тел.

    Б) Ускорение, приобретаемое телом, не объясняется действием на него других тел.

    В) Любая скорость, приобретаемая телом, объясняется действием на него других тел.

    Г) правильного ответа нет.

2. Мера инертных свойств тел называется . . .

   А) Силой.

   Б) Массой.

   В) Инерцией.

   Г) Силой трения.

3. Векторная физическая величина, характеризующая действие одного тела на другое, являющаяся причиной его деформации или изменения скорости, и определяемая произведением массы тела на ускорение его движения называется . . .

   А) Массой.

   Б) Инерцией.

   В) Силой.

   Г) Силой трения.

4. Физический смысл силы: сила …

    А) Показывает, на сколько изменяется скорость тела за единицу времени.

    Б) Численно равна единице, если телу массой 1 кг сообщено ускорение 1 м/с².

    В) Показывает, на сколько изменилось ускорение за единицу времени.

    Г) правильного ответа нет

5. Первый закон Ньютона утверждает, что . . .

    А) Скорость тела меняется при переходе из одной системы отчета в другую.

    Б) В инерциальной системе отчета скорость тела не меняется, если сумма сил, действующих на тело, равно нулю.

    В) Тела взаимодействуют с силами, равными по модулю, но противоположными по направлению.

    Г) На тело, погруженное в жидкость, действует выталкивающая сила.

6. Равнодействующая всех сил, действующая на тело, равна нулю, при этом тело …

    А) Движется равномерно прямолинейно.

    Б) Движется равномерно по окружности в горизонтальной плоскости.

    В) Находится в состоянии покоя.

    Г) Движется равномерно прямолинейно или находится в состоянии покоя.

7. Тело массой 20 кг, движущееся в инерциальной системе под действием силы 60 Н,   приобретает ускорение равное . . .

   А) 0,3 м/с².     Б) 40 м/с².     В) 3 м/с².     Г) 80 м/с².  

8.Два  мальчика с одинаковой массой тел взялись за руки. Первый мальчик толкнул  второго с силой 105 Н. Сила, с которой толкнул второй мальчик первого,  равна …

    А) 210 Н.        Б)  105 Н.     В) 50 Н.      Г) 0.

9. Пружина жесткостью  25 Н/м изменяет свою длину от 40 до 35 см под действием силы,  равной . . .

    А) 10 Н.      Б) 7,5 Н.     В) 5,25 Н.   Г) 1,25.

10. Динамометр с подвешенным грузом весом Р=3 Н свободно падает. Определите показания динамометра.

   А) 0 Н.      Б) 3 Н.       В) -3 Н.     Г) 9,8 Н.

Тест по физике на тему: «Динамика»

ТЕМА: «ДИНАМИКА»

(выбери правильный вариант ответа)

1. Кто открыл закон инерции?

А.      Аристотель;

Б.      Ломоносов;

В.      Ньютон;

Г.      Галилей.

2. Под действием силы 10 Н пружина длиной 1 м удлинилась на 0,1 м. Какова жёсткость пружины?

А.      10 Н/м;

Б.      100 Н/м;

В.      0,1 м/Н;

Г.      0,01 м/Н;

3. На тело действуют сила тяжести 30 Н и сила 40 Н, направленная горизонтально. Каково значение модуля равнодействующей этих сил?

А.      10 Н;

Б.      70 Н;

В.      50 Н;

Г.      1250 Н;

4. Под действием силы 10 Н тело получает ускорение 5 м/с2. Какова масса тела?

А.      2 кг;

Б.      0,5 кг;

В.      50 кг;

Г.      15 кг;

5. Одинаков ли вес одного и того же тела на экваторе и на полюсе Земли?

А.      Одинаков;

Б.      Неодинаков, больше на экваторе;

В.      Неодинаков, меньше на экваторе;

Г.      Зимой больше на экваторе, летом меньше на экваторе;

6. Под действием какой силы изменяется направление движения искусственных спутников, запущенных вокруг Земли?

А.      Силы трения;

Б.      Силы тяжести;

В.      Силы упругости;

Г.      Силы реакции опоры;

7. С какой силой упряжка собак равномерно перемещает сани с грузом массой 250 кг, если коэффициент трения скольжения 0,1?

А.      260 Н;

Б.      245 Н;

В.      25 Н;

Г.      250 Н;

8. Металлический шарик упал с некоторой высоты на металлическую плиту. От удара плита и шарик нагрелись. Подскочит ли шарик при отскоке вновь на ту же высоту?

А.      да, т.к. здесь справедлив закон сохранения механической энергии;

Б.      нет, он останется лежать на плите;

В.      нет, шарик подскочит на меньшую высоту, т.к. часть кинетической

энергии преобразовалась в тепловую, вызвавшую нагревание тел;

Г.      среди этих ответов нет правильного.

 

9. Что такое система отсчета?

А. система координат;
Б. прямоугольная система координат;
В. физическая величина;
Г. часы.
Д. верный ответ не приведен.

10. Как движется тело, если сумма всех действующих на него сил равна нулю?

А. неравномерно;
Б. прямолинейно;
В. с изменением скорости;
Г. прямолинейно и равномерно;
Д. равномерно по окружности.

11. От чего зависит сила тяжести?

А. ни от чего не зависит;
Б. от размеров тела;
В. от формы тела;
Г. от массы тела;
Д. от массы тела и величины g.

12. В каком пункте упомянуты только векторные физические величины?

А. скорость и путь;
Б. скорость и масса;
В. ускорение и время;
Г. сила и время.
Д. верный ответ не приведен.

Критерии оценивания.

  1. Ниже 40% правильных ответов на обязательную часть теста оценивается оценкой «2».

  2. Уровень 50% от числа вопросов на проверку знаний и умений на уровнях узнавания, воспроизведения и применения знаний оценивается «удовлетворительно» или 3 балла.

  3. За полное выполнение обязательной части теста, или за общее число правильных ответов, примерно соответствующее числу вопросов обязательной части теста, ставится оценка «хорошо» или 4 балла.

  4. Результату с превышением числа правильных ответов, над числом вопросов в обязательной части теста соответствует оценка «отлично», или 5 баллов. Такая оценка выставляется при полном овладении учебным материалом в соответствии с требованиями учебной программы на уровнях узнаваниях, воспроизведения и применения знаний в знакомой ситуации и обнаружении способности успешно применять полученные знания в незнакомой ситуации, способности творческого применения знаний при решении задач по физике.

Перевод числа правильных ответов на вопросы одного теста в оценку по пятибалльной шкале осуществляется по следующей шкале:

Количество правильных ответов

0-3

4-5

68

9-10

1112

Оценка

1

2

3

4

5

ОТВЕТЫ:

вопроса

1

2

3

4

5

6

7

8

9

10

11

12

Правильный ответ

Г

Б

В

А

В

Б

В

В

А

В

Б

Б

Тест Динамика (10 класс) с ответами по теме физики

Сложность: знаток.Последний раз тест пройден более 24 часов назад.

  1. Вопрос 1 из 10

    На полу лифта, движущегося вертикально вверх с ускорением, модуль которого равен а, лежит груз массой m. Чему будет равен модуль веса этого груза?

    • Правильный ответ
    • Неправильный ответ

    В вопросе ошибка?

    Следующий вопросПодсказка 50/50Ответить
  2. Вопрос 2 из 10

    Человек тянет за крючок динамометра с силой 50 Н, другой крючок прикреплён к стене.Каковы показания динамометра?

    • Правильный ответ
    • Неправильный ответ

    В вопросе ошибка?

    Подсказка 50/50Ответить
  3. Вопрос 3 из 10

    На столе лежит стопка книг массами 150г, 200г и 300г. Чему равна результирующая сила,действующая на нижнюю книгу ?

    • Правильный ответ
    • Неправильный ответ

    В вопросе ошибка?

    Подсказка 50/50Ответить
  4. Вопрос 4 из 10

    Кто открыл закон инерции?

    • Правильный ответ
    • Неправильный ответ

    В вопросе ошибка?

    Подсказка 50/50Ответить
  5. Вопрос 5 из 10

    Какова форма траектории космического корабля, которому сообщили первую космическую скорость?

    • Правильный ответ
    • Неправильный ответ

    В вопросе ошибка?

    Подсказка 50/50Ответить
  6. Вопрос 6 из 10

    Какие из трёх законов механики Ньютона выполняются только в инерциальных системах отсчёта?

    • Правильный ответ
    • Неправильный ответ

    В вопросе ошибка?

    Подсказка 50/50Ответить
  7. Вопрос 7 из 10

    Центростремительное ускорение определяется формулой

    • Правильный ответ
    • Неправильный ответ

    В вопросе ошибка?

    Подсказка 50/50Ответить
  8. Вопрос 8 из 10

    Тело брошено вертикально вниз с высоты 120 м со скоростью 10 м/с. Через сколько времени тело достигнет поверхности Земли?

    • Правильный ответ
    • Неправильный ответ

    В вопросе ошибка?

    Подсказка 50/50Ответить
  9. Вопрос 9 из 10

    Сила — причина

    • Правильный ответ
    • Неправильный ответ

    В вопросе ошибка?

    Подсказка 50/50Ответить
  10. Вопрос 10 из 10

    Как изменится величина скорости искусственного спутника Земли при увеличении радиуса его орбиты?

    • Правильный ответ
    • Неправильный ответ

    В вопросе ошибка?

    Подсказка 50/50Ответить

Доска почёта

Чтобы попасть сюда — пройдите тест.

ТОП-3 тестакоторые проходят вместе с этим

Тест «Динамика» (10 класс) с ответами является прекрасным помощником в процессе домашней подготовки к урокам. Большинство вопросов посвящено умению решать задачи по теме, некоторые направлены на проверку теоретических знаний. Тест включает вопросы разного уровня сложности, что позволяет объективно оценить свои знания. Прохождение его подразумевает выбор одного варианта из нескольких, поэтому выполнение всех заданий не займет много времени.

Тест по физике «Динамика» поможет самостоятельно повторить и систематизировать материал для успешного написания текущих и итоговых проверочных работ.

Рейтинг теста

Средняя оценка: 3.3. Всего получено оценок: 437.

А какую оценку получите вы? Чтобы узнать — пройдите тест.

Тест по физике 10 класс «динамика» | Тест по физике (10 класс) по теме:

Вариант 1.

№1. В механике сила обозначается

        1)  R,                2) t,                 3) a,                 4) F.

№2. В механике единицей измерения ускорения является

        1)  м/с,                2) м/с2,                 3) м2/с,                 4) м2.

№3.  Формула, выражающая второй закон Ньютона

        1)  F=ma,                2) F=mg,                 3) ,                 4) .

№4. Сила притяжения яблока к Земле равна 2 Н. С какой по модулю силой яблоко притягивает к себе Землю?

        1)  2 Н,                2) -2 Н,                 3) 0 Н,                 4) 20 Н.

№5. Сила всемирного тяготения зависит

        1)  от ускорения свободного падения,        2) только от массы тел,         

3) от массы тел и расстояния между ними,         4) от среды, в которую помещены тела.

№6. Утверждение, что материальная точка покоится или движется равномерно прямолинейно, если на нее не действуют другие тела или действие других тел скомпенсировано:

        1)  верно при любых условиях,        2) верно для инерциальных систем отсчета,

3) верно для неинерциальных систем отсчета,         4)неверно ни для каких систем отсчета.

№7. На левом рисунке представлены вектор скорости и вектор равнодействующей всех сил, действующих не тело. Какой из векторов на правом рисунке указывает направление вектора ускорения этого тела в инерциальных системах отсчета?

        1)  1,                2) 2,                 3) 3,                 4) 4.

№8. Космонавт, находясь на Земле, притягивается к ней с силой 700 Н. С какой приблизительно силой он будет притягиваться к Марсу, находясь на его поверхности? Радиус Марса в 2 раза, а масса – в 10 раз меньше чем у Земли.

        1)  70 Н,                2) 140 Н,                 3) 210 Н,                 4) 280 Н.

№9. Мальчик массой 50 кг совершает прыжок в высоту. Сила тяжести, действующая на него во время прыжка примерно равна

        1)  500 Н,                2) 50 Н,                 3) 5 Н,                 4) 0 Н.

№10. Тело равномерно движется по плоскости. Сила давления тела на плоскость равна 20 Н, сила трения 5 Н. Коэффициент трения скольжения равен

        1)  0,8,                2) 0,25,                 3) 0,75,                 4) 0,2.

А1

А2

А3

А4

А5

А6

А7

А8

А9

А10

1

х

х

2

х

х

х

3

х

х

4

х

х

х

B1

С  вершины  наклонной  плоскости  из  состояния  покоя скользит  с  ускорением  лёгкая  коробочка,  в  которой находится  груз  массой  m  (см.  рисунок).  Как  изменятся время движения, ускорение и модуль работы силы трения, если с той же наклонной плоскости будет скользить та же

коробочка с грузом массой 2m?

Для каждой величины определите соответствующий характер изменения:  

1)  увеличится

2)  уменьшится

3)  не изменится

 

Запишите в  таблицу  выбранные  цифры  для  каждой физической  величины.

Цифры в ответе могут повторяться.

3 3 1

С 2

Грузы  массами  M = 1 кг и  m  связаны

лёгкой  нерастяжимой  нитью,

переброшенной через блок, по которому

нить  может  скользить  без  трения ( см.

рисунок). Груз  массой M  находится на

шероховатой  наклонной  плоскости

(угол  наклона  плоскости  к  горизонту

α = 30°, коэффициент трения μ = 0,3). Чему равно максимальное значение

массы m, при котором система грузов ещё не выходит из первоначального

состояния покоя? Решение поясните схематичным рисунком с  указанием

используемых сил.

С3 Масса Марса составляет 0,1 от массы Земли, диаметр Марса вдвое меньше диаметра Земли. Чему равно отношение периодов обращения искусственных спутников Марса и Земли, движущихся по круговым орбитам на небольшой высоте?

Вариант 2.

№1. В механике коэффициент трения обозначается

        1)  R,                2) ,                 3) a,                 4) F.

№2. В механике единицей измерения силы является

        1)  м/с,                2) м/с2,                 3) Н,                 4) кг.

№3.  Формула, выражающая закон всемирного тяготения

        1)  F=ma,                2) F=mg,                 3),                 4) .

№4. Сила притяжения яблока к Земле равна 2 Н. С какой по модулю силой яблоко притягивает к себе Землю?

        1)  2 Н,                2) -2 Н,                 3) 0 Н,                 4) 20 Н.

№5. Сила трения скольжения

        1)  зависит  от скорости движения тела,        2) зависит  от веса тела и рода поверхности,         

3) зависит  от веса тела и площади поверхности,         4) не зависит ни от чего.

№6. Систему отсчета, связанную с Землей, будем считать инерциальной. Система отсчета, связанная с автомобилем, тоже будет инерциальной, если автомобиль

        1)  движется равномерно по прямолинейному участку шоссе,        

2) разгоняется по прямолинейному участку шоссе,

3) движется равномерно по извилистой дороге,

4) по инерции вкатывается на гору.

№7. На левом рисунке представлены вектор скорости и вектор ускорения тела. Какой из векторов на правом рисунке указывает направление вектора равнодействующей всех сил, действующих на это тело?

        1)  1,                2) 2,                 3) 3,                 4) 4.

№8. Космическая ракета удаляется от Земли. На каком расстоянии от земной поверхности сила гравитационного притяжения ракеты Землей уменьшится в 4 раза по сравнению с силой притяжения на земной поверхности? (Расстояние выражается в радиусах Земли R.)

        1)  R,                2) R,                 3) 2R,                 4) 3R.

№9. На рисунке представлен график зависимости силы упругости пружины от величины ее деформации. Жесткость этой пружины равна

        

1)  10 Н/м,                2) 20 Н/м,                 3) 100 Н/м,                 4) 0,01 Н/м.

№10. Конькобежец массой 70 кг скользит по льду. Какова сила трения, действующая на конькобежца, если коэффициент трения скольжения коньков по льду равен 0,02?

        1)  0,35 Н,                2) 1,4 Н,                 3) 3,5 Н,                 4) 14 Н.

А1

А2

А3

А4

А5

А6

А7

А8

А9

А10

1

х

х

х

2

х

х

х

3

х

х

4

х

х

В1

В  результате  перехода  с  одной  круговой  орбиты  на  другую центростремительное  ускорение  спутника  Земли  уменьшается.  Как изменяются в результате этого  перехода радиус  орбиты спутника, скорость его движения по орбите и период обращения вокруг Земли?

Для каждой величины определите соответствующий характер изменения:  

1)  увеличилась

2)  уменьшилась

3)  не изменилась

Запишите  в таблицу выбранные цифры  для  каждой  физической величины.

Цифры в ответе могут повторяться.

1 2 1

С2.

По горизонтальной дороге мальчик тянет сани массой 30 кг за веревку, направленную под углом 600 к плоскости дороги, с силой 100 Н. Коэффициент трения 0,12. Определите ускорение саней. Каков путь, пройденный санями за 5 с, если в начальный момент времени их скорость была равна нулю?

С3.

Грузики с точечными массами m1=0,25кг и  m2=0,5кг прикреплены к невесомому стержню длиной 1 м. Стержень может вращаться вокруг горизонтальной оси, проходящей через точку О. Грузик m2 в нижней точке траектории имеет скорость 2 м/с. Определите силу, с которой стержень действует на грузик m1 в этот момент времени.

Вариант 3.

№1. В механике жесткость пружины обозначается

        1)  R,                2) ,                 3) k,                 4) F.

№2. В механике единицей измерения жесткости является

        1)  Н/м,                2) м/с2,                 3) Н,                 4) м/Н.

№3.  Формула, выражающая третий закон Ньютона

        1)  F=ma,                2) ,                 3) ,                 4) F= — kx.

№4. Известно, что масса Земли примерно в 81 раз больше массы Луны. Чему равно отношение силы, с которой Земля притягивает к себе Луну к силе, с которой Луна притягивает к себе Землю?

        1)  81,                2) 9,                 3) 1,                 4) 1/81.

№5. Жесткость пружины

        1)  зависит  от силы упругости,        2) зависит  от удлинения пружины,         

3) не зависит ни от чего,         4) зависит от длины пружины.

№6. Самолет летит по прямой с постоянной скоростью на высоте 9000 м. Систему отсчета, связанную с Землей, будем считать инерциальной. В этом случае

        1)  на самолет не действует сила тяжести,        

2) сумма всех сил, действующих на самолет, равна нулю,

3) на самолет не действуют никакие силы,

4) сила тяжести равна силе Архимеда, действующей на самолет.

№7. Какая из приведенных пар величин всегда совпадает по направлению?

        1)  сила и ускорение,         2) сила и скорость,

        3) сила и перемещение,         4) ускорение и перемещение.

№8. Во сколько раз сила притяжения Земли к Солнцу больше силы притяжения Меркурия к Солнцу? Масса Меркурия составляет 1/18 массы Земли, а расположен он в 2,5 раза ближе к Солнцу, чем Земля.

        1)  в 2,25 раза,        2) в 2,9 раз,           3) в 7,5 раз,                 4) в 18 раз.

№9. Под действием силы 3 Н пружина удлинилась на 4 см. Чему равен модуль силы, под действием которой удлинение этой пружины составит 6 см?

1)  3,5 Н,                2) 4 Н,                 3) 4,5 Н,                 4) 5  Н.

№10. Тело равномерно движется по плоскости. Сила давления тела на плоскость равна 20 Н, сила трения 5 Н. Коэффициент трения скольжения равен

        1)  0,8,                2) 0,25,                 3) 0,75,                 4) 0,2.

А1

А2

А3

А4

А5

А6

А7

А8

А9

А10

1

х

х

2

х

х

х

х

3

х

х

х

х

4

Вариант 4.

№1. В механике гравитационная постоянная обозначается

        1)  R,                2) ,                 3) k,                 4) G.

№2. В механике коэффициент трения измеряется

        1) в м/с,                2) в Н/м,                 3)в Н,                 4) является безразмерной величиной.

№3.  Формула, выражающая закон Гука

        1)  F=ma,                2) F=mg,                 3) F= -kx,                 4) .

№4. Сила притяжения яблока к Земле равна 2 Н. С какой по модулю силой яблоко притягивает к себе Землю?

        1)  2 Н,                2) -2 Н,                 3) 0 Н,                 4) 20 Н.

№5. Гравитационная постоянная

        1)  зависит  от скорости движения тел,        2) зависит  от массы тел и расстояния между ними,         3) зависит  от выбора системы отсчета,         4) не зависит ни от чего.

№6. Парашютист спускается по вертикали с постоянной скоростью 2 м/с. Систему отсчета, связанную с Землей, будем считать инерциальной. В этом случае

        1)  на парашютиста не действуют никакие силы,        

2) сила тяжести равна нулю,

3) сумма всех сил, приложенных к парашютисту равна нулю,

4) сумма всех сил, приложенных к парашютисту постоянна и не равна нулю.

№7. На левом рисунке представлены вектор скорости и вектор ускорения тела. Какой из векторов на правом рисунке указывает направление вектора равнодействующей всех сил, действующих на это тело?

        1)  1,                2) 2,                 3) 3,                 4) 4.

№8. Космический корабль движется вокруг Земли по круговой орбите радиусом 2.107 м. Его скорость  равна

        1)  4,5 км/с,                2) 6,3 км/с,                 3) 8 км/с,                 4) 11 км/с.

№9. Ученик собрал установку, используя нить, пружину и штатив. Деформация пружины 0,05 м, ее жесткость равна 40 Н/м. Сила натяжения нити равна

        

1)  800 Н,                2) 0,05 Н,                 3) 2 Н,                 4) 0 Н.

№10. Конькобежец массой 70 кг скользит по льду. Какова сила трения, действующая на конькобежца, если коэффициент трения скольжения коньков по льду равен 0,02?

        1)  0,35 Н,                2) 1,4 Н,                 3) 3,5 Н,                 4) 14 Н.

А1

А2

А3

А4

А5

А6

А7

А8

А9

А10

1

х

2

3

х

х

х

х

4

х

х

х

х

х

1

2

3

4

4

3

2

1

4

3

2

1

4

3

2

1

О

l/2

m2

l/2

m1

v

Контрольный тест по теме Динамика. Физика. 10 класс.

Методическая разработка: «Контрольный тест по теме «Динамика» к учебнику В.А. Касьянов «Физика. 10 класс»

Автор разработки: Повещенко Ольга Константиновна, преподаватель физики ГБПОУ КК ЕПК, г. Ейск.

Контрольный тест по теме «Динамика» к учебнику В.А. Касьянов «Физика. 10 класс»

Вариант 1

Движение по инерции – это:

Движение тела, происходящее без внешних воздействий+

Движение тела без ускорения

Движение с периодическим внешним воздействием

Тело может двигаться:

Только при наличии внешнего воздействия

Как при наличии, так и при отсутствии внешнего воздействия+

Только при наличии силы тяги

Коэффициент пропорциональности между ускорением и силой, действующей на тело:

Скорость

Масса +

Импульс

Силы, с которыми два тела действуют друг на друга:

Равны по модулю, противоположны по направлению и действуют вдоль одной прямой+

Равны по модулю, одинаковы по направлению и действуют вдоль одной прямой

Пропорциональны их массам, противоположны по направлению и действуют вдоль одной прямой

Сила упругости является проявлением:

Гравитационного взаимодействия

Сильного взаимодействия

Электромагнитного взаимодействия+

Сила упругости зависит:

От изменения расстояния между частями одного и того же тела+

От расстояния между различными телами

От расстояния между частями одного и того же тела

Гравитационная сила притяжения между двумя телами:

Пропорциональна произведению их масс и обратно пропорциональна квадрату расстояния между ними+

Пропорциональна квадрату расстояния между ними и обратно пропорциональна произведению их масс

Пропорциональна произведению масс и обратно пропорциональна расстоянию между ними

К чему приложен вес?

К телу

К опоре

К опоре или подвесу+

Сила трения направлена:

Вдоль поверхности соприкосновения+

Перпендикулярно плоскости соприкосновения

Под углом к плоскости соприкосновения

Сила упругости пропорциональна удлинению тела:

При любых деформациях

При малых деформациях +

При больших деформациях

Тело массой 40 кг движется под действием постоянной силы 10 Н. Выберите верную характеристику движения:

Равноускоренное с ускорением 4 м/с2

Равномерное со скоростью 4 м/с

Равноускоренное с ускорением 0.25 м/с2+

Мальчик массой 60 кг действует силой 12 Н на тележку массой 12 кг. С какой силой тележка действует на мальчика?

1 Н

5 Н

12 Н+

Под действием силы 8 Н пружина удлинилась на 0.04 м. Чему равна жесткость пружины?

200 Н/м+

0.32 Н/м

0.005 Н/м

На расстоянии R от поверхности Земли на тело действует сила всемирного тяготения 6 Н. Чему равна сила тяготения, если расстояние уменьшить вдвое?

3 Н

12 Н

24 Н +

Под действием силы 300 Н по горизонтальной поверхности равномерно тянут груз. Найти массу груза, если коэффициент трения 0.5.

600 кг

60 кг +

150 кг

Вариант 2

Законом инерции иначе называют:

Первый закон Ньютона +

Второй закон Ньютона

Третий закон Ньютона

Мерой взаимодействия тел является:

Скорость

Масса

Сила+

Количественной мерой инертности является:

Масса+

Сила

Скорость

Равнодействующая сила – это

Векторное произведение сил, действующих на тело

Векторная сумма сил, действующих на тело+

Сумма модулей сил, действующих на тело

Сила упругости направлена:

Вдоль смещения частиц при деформации

Перпендикулярно смещению частиц при деформации

Против смещения частиц при деформации+

Сила трения является проявлением:

Гравитационного взаимодействия

Сильного взаимодействия

Электромагнитного взаимодействия+

Какие виды трения возможны при контакте твердых тел?

Трение качения, трение скольжения, трение покоя+

Трение качения, трение скольжения

Трение качения, трение скольжения, трение покоя, трение кручения

Чему равен вес тела в состоянии невесомости?

Силе тяжести

Нулю+

Силе гравитационного взаимодействия

Какое утверждение является верным?

Максимальная сила трения покоя не зависит от площади соприкасающихся поверхностей+

Максимальная сила трения покоя зависит от площади соприкасающихся поверхностей

Максимальная сила трения покоя не зависит от площади соприкасающихся поверхностей и силы нормального давления

Третий закон Ньютона справедлив для:

Только гравитационного и электромагнитного взаимодействия

Любого фундаментального взаимодействия+

Только сильного и слабого взаимодействия

Тело массой 5 кг движется под действием постоянной силы 10 Н. Выберите верную характеристику движения:

Равноускоренное с ускорением 0.5 м/с2

Равномерное со скоростью 2 м/с

Равноускоренное с ускорением 2 м/с2+

Мальчик массой 50 кг действует силой 10 Н на тележку массой 10 кг. С какой силой тележка действует на мальчика?

2 Н

10 Н+

6 Н

Под действием силы 10 Н пружина удлинилась на 0.05 м. Чему равна жесткость пружины?

200 Н/м+

0.5 Н/м

0.005 Н/м

На расстоянии R от поверхности Земли на тело действует сила всемирного тяготения 16 Н. Чему равна сила тяготения, если расстояние увеличить вдвое?

32 Н

8 Н

4 Н +

Под действием силы 600 Н по горизонтальной поверхности равномерно тянут груз. Найти коэффициент трения, если масса груза 120 кг.

5

0.5 +

0.2

1

2

3

4

5

6

7

8

9

10

11

12

13

14

15

В 1

1

2

2

1

3

1

1

3

1

2

3

3

1

3

2

В 2

1

3

1

2

3

3

1

2

1

2

3

2

1

3

2

Адрес публикации: https://www.prodlenka.org/metodicheskie-razrabotki/389362-kontrolnyj-test-po-teme-dinamika-fizika-10-kl

Тесты и контрольные работы

Электронное тестирование по теме «Термодинамика» 10 класс. Выполнено на основе Web технологий. В архиве html файл и flash ролик. Для проигрывания нужен Flash плеер. Запускаем html файл и отвечаем на поставленные вопросы. По окончании выполнения заданий, программа покажет результат и учитель может оценить работу.

                                                           Тест по теме «Динамика» 10 класс   Тест по физике «Динамика» предназначена для учителей физики и учащихся средних школ. Учитель может использовать  на уроке при закреплении  материала по теме урока. Учащиеся также могут её использовать при изучении темы «Динамика» самостоятельно, при повторении, закреплении своих знаний и для их проверки.

                                         Контрольная работа по теме «Кинематика» 10 класс

Контрольная работа № 1 по теме «Основы кинематики» — 10 класс. Содержит 24 варианта. 
                                  «Основы молекулярно-кинетической теории» — тесты 10 класс

Электронные тесты по теме «Основы МКТ» позволяют проверить знание таких тем, как основное уравнение МКТ, знание основных постоянных величин и их единиц измерения, понятия температуры. Тесты предназначены для учащихся 10 класса, учебник физика 10, автор Г.Я. Мякишев. Шаблоны тестов взяты из интернета.

 Данные материалы по теме «Электромагнитные явления» позволяют определить уровень усвоения  обучающихся содержат различные по сложности и форме задание: выбор ответа из предложенных вариантов, подстановка формул, заполнение таблицы, запись определений, дат и имён ученых, решение задач. Предложены варианты решения задач.

 Тест состоит из 50 вопросов с ответами, на тему Электромагнитные колебания. Данную работу может использовать не только учитель в школе для работы на уроке физики или на факультативах, но и ученик дома для самопроверки собственных знаний и самоподготовки к государственному экзамену по физике. Каждый вопрос теста имеет 5 вариантов ответа,причем и вопросы и варианты ответа выдается в случайном порядке, и повторение варианта невозможно. Это сделано для того, чтобы ученик не мог механически запомнить правильные ответы. После прохождения теста появляется статистика и оценка.Результат тестирования учитель может сохранить в электронном варианте или распечатать, а ученик может проверить в каких вопросах сделал ошибки.

 

 

Тест по физике Динамика свободных и вынужденных колебаний для 10 класса

Тест по физике Динамика свободных и вынужденных колебаний для 10 класса с ответами. Тест включает в себя 2 варианта. В каждом варианте по 5 заданий.

1 вариант

1. Как изменится период колебаний математического маятника, если длину нити увеличить в 4 раза?

А. Увеличится в 2 раза.
Б. Уменьшится в 2 раза.
В. Увеличится в 4 раза.

2. Как изменится период колебаний пружинного маятни­ка, если жесткость пружины уменьшить в 9 раз?

А. Увеличится в 9 раз.
Б. Уменьшится в 3 раза.
В. Увеличится в 3 раза.

3. Как изменяется полная механическая энергия гармо­нических колебаний пружинного маятника при увеличе­нии амплитуды его колебаний в 2 раза?

А. Не изменяется.
Б. Увеличивается в 4 раза.
В. Уменьшается в 2 раза.

4. Груз висит на пружине и колеблется с периодом 0,6 с. На сколько укоротится пружина, если снять с нее груз?

А. На 3 см.
Б. На 9 см.
В. На 15 см.

5. Какова зависимость амплитуды вынужденных колеба­ний от частоты, если амплитуда колебаний вынуждаю­щей силы постоянна?

А. Не зависит от частоты.
Б. Непрерывно возрастает с увеличением частоты.
В. Сначала возрастает, достигает максимума, а затем убывает.

2 вариант

1. Как изменится период колебаний пружинного маятни­ка, если массу маятника уменьшить в 9 раз?

А. Увеличится в 3 раза.
Б. Уменьшится в 3 раза.
В. Не изменится.

2. Как изменится период колебаний математического ма­ятника, если длину нити увеличить в 4 раза?

А. Увеличится в 4 раза.
Б. Уменьшится в 4 раза.
В. Уменьшится в 2 раза.

3. Как изменяется полная механическая энергия гармо­нических колебаний пружинного маятника при умень­шении амплитуды его колебаний в 3 раза?

А. Не изменяется.
Б. Увеличивается в 3 раза.
В. Уменьшается в 9 раз.

4. Каково растяжение пружины, жест­кость которой равна 200 Н/м, под дейст­вием подвешенного груза массой 2 кг?

А. 10 см.
Б. 20 см.
В. 5 см.

5. На рисунке 25 представлены резо­нансные кривые.

Какая кривая соответ­ствует наименьшему значению силы тре­ния в системе?

Ответы на тест по физике Динамика свободных и вынужденных колебаний для 10 класса
1 вариант
1-А
2-В
3-Б
4-Б
5-В
2 вариант
1-В
2-В
3-В
4-А
5-А

Модуль 2 Практический тест «Динамика» | 11 класс физики

Практический тест для 11 класса физики Модуль 2 «Динамика»

Сдаете ли вы раз в полгода экзамен по динамике модуля 2 физики 11 класса?

Мы собрали 10 обязательных экзаменационных вопросов по модулю 2 физики 11 класса «Динамика», которые помогут вам успешно сдать полугодовой экзамен.

Модуль 2 «Динамика» оценивает фундаментальные законы физики, включая законы Ньютона, и считается самым важным модулем по физике 11-го года обучения.

Модуль 2 «Динамика» Практический тест по физике охватывает следующие темы:

  • Законы Ньютона
  • Силы, действующие на отдельные тела
  • Анализ сил в двух измерениях
  • Анализ сил на наклонных склонах
  • Статическое трение
  • Энергия и работа
  • График импульса-времени
  • Импульс и импульс
  • Столкновения

Связано: Ищете учебный тест по физике 11-го года обучения по модулю 1 «Кинематика»?

Год 11 Физика Практический тест модуля 2 «Динамика»

Инструкции

Общие инструкции для сдачи практического теста Модуля 2 «Динамика» изложены ниже.

  • Время чтения: 5 минут
  • Время работы: 55 минут
  • Всего оценок: 31 балл
  • Напишите черной ручкой
  • Нарисуйте диаграммы карандашом
  • Можно использовать калькуляторы, одобренные NESA
  • Можно использовать формулу, таблицу данных и таблицу Менделеева.
  • Есть 10 вопросов с оценками от 1 до 5 баллов.

На заключительном экзамене HSC по физике у вас будет примерно 1.8 минут на отметку.

На практических экзаменах вы должны стремиться отвечать на вопросы с шагом 1,5 минуты на каждую оценку.

Для имитации реальных условий экзамена:

  • Уберите заметки и другие отвлекающие факторы в учебную среду
  • Закройте все ненужные вкладки и отключите уведомления на своем устройстве
  • Установите таймер на 55 минут
  • Завершите этот экзамен в тихой обстановке, где вас никто не побеспокоит во время экзамена.{-1} через 6 секунд.

    Какая чистая внешняя сила действует на автомобиль? (1 балл)

    (A) 816 N
    (B) 8000 N
    (C) 12000 N
    (D) 9010 N

    См. Решение вопроса 1.

    Вопрос 2: Законы Ньютона

    Автомобиль мчится по дороге в заснеженных горах и выезжает на ледяной поворот.{-1} через 7,5 секунд.

    (а) Какое было ускорение автомобиля? 1
    (b) Какая чистая внешняя сила действовала на автомобиль? 1
    (c) Движущая сила двигателя автомобиля составляла 1600 Н. Если единственной другой действующей силой было трение дороги, какова была величина этой силы трения? 2

    См. Решение вопроса 3.

    Вопрос 4: Анализ сил в двух измерениях

    Автомобиль движется по склону 5 \ градуса с постоянной скоростью. Силы трения, действующие на автомобиль, равны 1000 Н.

    (a) Нарисуйте диаграмму, показывающую все силы, действующие на автомобиль. Четко обозначьте силы. 1
    (b) Определите чистую силу, действующую на транспортное средство в наклонной плоскости. 1
    (в) Определите массу автомобиля.Выразите свой ответ четырьмя значащими цифрами. 2

    См. Решение вопроса 4.

    Вопрос 5: Анализ сил в шкивных системах.

    Два блока весом 2 кг и 4 кг находятся на двойных наклонных склонах, как показано ниже.

    (a) Рассчитайте ускорение блоков. Выразите свой ответ двумя значащими цифрами. 2
    (b) Отсюда или иначе рассчитайте натяжение струны. 2

    См. Решение вопроса 5.

    Вопрос 6: Статическое трение

    Мэдисон проводит эксперимент по вычислению коэффициента статического трения между контактными поверхностями — наждачной бумагой и деревом. Она ставит эксперимент, как показано на диаграмме ниже.

    Масса деревянного бруска с приклеенной с одной стороны наждачной бумагой составила 0,25 кг.

    Она потянула пружинный баланс в горизонтальном направлении до тех пор, пока деревянный брусок не соскользнул, и записала показание силы балансировки пружины в тот момент, когда это произошло.Затем она добавила 1, 2, 3 и 4 дополнительные 100-граммовые гири поверх блока, записав по 3 попытки для каждой гири. Ее результаты показаны ниже.

    Масса (кг) Приложенная сила (Н)
    испытательная 1
    Приложенная сила (Н)
    пробная 2
    Приложенная сила (Н)
    испытание 3
    0,25 1,6 2,0 2.0
    0,35 3,2 3,4 2,0
    0,45 3,3 3,8 4,0
    0,55 5,0 6,3 4,3

    (a) Постройте график зависимости максимального статического трения от массы 2
    (b) Определите значимость градиента. 2
    (c) Так или иначе, определите коэффициент статического трения для поверхности наждачной бумаги и дерева. (4 балла) 2

    См. Решение вопроса 6.

    Вопрос 7: Энергия и работа

    Floyd тянет 60-килограммовую шину с усилием 250 Н под углом 20 градусов к горизонтали.

    (a) Если он может тянуть 25 метров, рассчитайте работу, выполненную Флойдом.{-1} \ Север.

    a) Каков начальный импульс автомобиля? 1
    б) Каков начальный импульс грузовика? 1
    c) Если автомобиль и грузовик соединяются в результате столкновения, какова их конечная скорость? Обязательно укажите величину и направление. Выразите свой ответ тремя значащими цифрами. 2

    См. Решение вопроса 10.{-2} \ end {align}

    Шаг 3: Рассчитайте чистую внешнюю силу, действующую на автомобиль, по формуле F net = ma

    \ begin {align} F_ {net} & = ma \\ & = 2400 \ times \ dfrac {20} {6} \\ & = 8000 \ N \ end {выровнено}

    Ответ: (B)

    Вернуться к вопросу 1.

    Вопрос 2 решение

    Отвечая на вопросы «объяснения», учащиеся должны использовать систему Learnable CEO Framework TM (Причина, Следствие, Результат) для построения логического и последовательного ответа.

    Структура CEO Деталь
    Причина Первый закон Ньютона гласит, что объект будет продолжать движение, если на него не будет воздействовать внешняя сила.
    Эффект Следовательно, когда автомобиль движется по изгибу, трение между его шинами и дорогой создает чистую центростремительную силу (внешнюю силу), которая позволяет автомобилю совершать круговое движение, тем самым огибая поворот.Однако на обледенелой дороге трение между шинами и дорогой практически отсутствует.
    Результат Следовательно, на автомобиль не действует никакая внешняя сила, и, согласно Первому закону Ньютона, автомобиль продолжает двигаться прямо вперед с исходной скоростью и не может повернуть за угол.

    Таким образом, наш логический и последовательный ответ приведен ниже:

    Первый закон Ньютона гласит, что объект будет продолжать движение в своем состоянии, если на него не будет действовать внешняя сила.Следовательно, когда автомобиль движется по повороту, трение между его шинами и дорогой создает чистую центростремительную силу (внешнюю силу), которая позволяет автомобилю совершать круговое движение, тем самым огибая поворот. Однако на обледенелой дороге трение между шинами и дорогой практически отсутствует. Следовательно, на автомобиль не действует никакая внешняя сила, и, согласно Первому закону Ньютона, автомобиль продолжает двигаться прямо вперед со своей первоначальной скоростью и не может повернуть за угол.

    Вернуться к вопросу 2.{-2}

    Часть (b)

    Из Второго закона Ньютона:

    \ begin {align} F_ {net} & = m \ vec {a} \\ & = (540) (2) \\ & = 1080 \ N \ end {align}

    Ответ : 1080 \ N

    Часть (c)

    Нарисовав схему автомобиля и сил, действующих на него, мы можем чтобы увидеть, как силы взаимодействуют друг с другом:

    Следовательно, чистую силу, действующую на автомобиль, можно найти, оценив действующие на него горизонтальные силы:

    \ begin {align} F_ {net} & = driving \ force — трение \ сила \\ & = 1600 — f \\ \, следовательно, f & = 1600 — F_ {net} \ end {align}

    Из части (b), мы теперь, что чистая сила, действующая на автомобиль, составляет 1080 \ N.Подставляем урожайность:

    f = 1600 — 1080 = 520 \ N

    Ответ : 520 \ N

    Вернуться к вопросу 3.

    Решение вопроса 4

    Часть (a)

    Нарисованная диаграмма должна отображать нормальную силу реакции, силу веса и силу трения:

    Часть (b)

    Чистая сила, действующая вдоль склона, должна быть ноль , если скорость спуска остается постоянной.

    Ответ: 0

    Часть (c)

    Поскольку автомобиль движется с постоянной скоростью, силы сопротивления, действующие вверх по склону, должны уравновешивать нисходящую составляющую силы веса:

    \ begin { выровнено} mg sin \ theta & = 1000 \ N \\\\ m & = \ frac {1000} {g sin \ theta} \\\\ m & = \ frac {1000} {9.8 sin 5 \ deg} \\ \\ \ поэтому m & = 1171 \ kg \ (4 \ sf) \ end {выровнено}

    Ответ : 1171 \ kg

    Вернуться к вопросу 4.

    Вопрос 5

    Часть (a)

    Шаг 1. Нарисуйте диаграмму свободного тела каждого объекта (включая внутренние силы), чтобы найти выражение чистой силы, действующей на каждый объект.

    Шаг 2: Найдите чистую внешнюю силу на всю систему и примените второй закон Ньютона, чтобы найти ускорение.

    Внешние силы, действующие на блоки:

    1. Составляющая веса блока 1, , опирающаяся на наклон \ theta = 30 \ градус: 2gsin30 \ градус
    2. Составляющая веса блока 2 опирается на наклон \ theta = 45 \ градус: 4gsin45 \ градус

    Мы можем «развернуть» эту проблему так, чтобы она выглядела как блоки, соединенные нитью.{-2}

    Часть (b)

    Шаг 1: Добавьте силы, действующие на блок 2 кг

    \ begin {align} \ Sigma F & = ma \\ T — 2gsin45 \ deg & = 2a \\ T = 2gsin45 \ deg & + 2a \ end {align}

    Шаг 2: подставьте значение ускорения в уравнение натяжения, чтобы найти значение T.

    \ begin {align} T & = 2gsin45 \ deg + 2a \\ & = 2 \ times 9,8 sin 45 \ deg + 2 \ times 3.0 \\ & = 19.859 … \\ & = 19.9 \ N \ (3 \ sf) \ end {выровнено}

    Ответ : 19.9 \ N

    Вернуться к вопросу 5.

    Решение вопроса 6

    Часть (a) Чтобы построить график зависимости максимального статического трения от массы, нам нужно найти среднюю приложенную силу, которая вызывает скольжение массы.

    Шаг 1. Определите максимальное статическое трение (f_ {s (max)}) на массу для каждой комбинации поверхностей.

    Масса (кг) Среднее приложенное усилие (Н) f_ {s (макс)}
    0.25 1,9 1,9
    0,35 2,9 2,9
    0,45 3,7 3,7
    0,55 4,7 4,7 5.2

    Шаг 2: Постройте график результатов, полученных на шаге 1, показывая взаимосвязь между f_ {s (max)} и независимой переменной. Нарисуйте на графике линию, наиболее подходящую для вас.

    График зависимости f_ {s (max)} от массы (м) для поверхности наждачной бумаги и дерева показан ниже:

    Часть (b): Чтобы определить значимость градиента, измените уравнение f_ {s (max)} = \ mu_s mg в форме y = mx.

    Дано

    • f_ {s (max)} = \ mu_s N = \ mu_s mg
    • f_ {s (max)} = \ mu_s mg
    • \ поэтому f_ {s (max)} = (\ mu_sg) \ times m

    График зависимости f_ {s (max)} от массы (m) является линейным.

    • f_ {s (max)} — зависимая переменная (ось y)
    • m — независимая переменная (ось x)
    • \ поэтому gradient = (\ mu_sg)

    Part (c )

    Шаг 1: Рассчитайте градиент линии наилучшего соответствия.

    \ begin {align} gradient & = \ frac {rise} {run} \\ & = \ frac {4.6 — 2.2} {0.56 — 0.28} \\ & = 8.6 \ end {align}

    Шаг 2 : Используя результаты, полученные в (b), рассчитайте коэффициент трения покоя.

    Мы можем вывести взаимосвязь между коэффициентом статического трения и градиентом линии наилучшего соответствия:

    \ begin {align} \ следовательно gradient & = \ mu_s g \\ \ следовательно \ mu_s & = \ frac { gradient} {g} \\ & = \ frac {gradient} {9.8} \\ & = \ dfrac {8.6} {9.8} \\ & = 0.88 \ end {align}

    Ответ: 0.88

    Назад к Вопрос 6.

    Решение вопроса 7

    Часть (a)

    Прежде чем использовать формулу для расчета проделанной работы, мы должны найти компонент силы в направлении смещения F_x.Это связано с тем, что сила в направлении y не влияет на горизонтальное движение шины.

    \ begin {выровнен} W & = F_xd \\ & = (250 cos 20 \ градус) (25) \\ & = 5873,07 … \ J \ end {выровнен}

    Ответ : 5900 \ J ( 2 sf)

    Часть (b)

    Шаг 1: Рассчитайте работу, необходимую для преодоления силы трения .

    Чтобы вывести шину из ее неподвижного положения, необходимо выполнить работу по преодолению силы трения между шиной и землей.Объем работы, необходимой для преодоления этой силы трения, определяется следующим образом:

    \ begin {выровнено} W_f & = F_fd \\ & = (220) (25) \\ & = 5500 \ J \ end {align}

    Шаг 2: Рассчитайте кинетическую энергию, которую Флойд обеспечивает шине.

    Следовательно, компонент работы, выполняемой Флойдом по обеспечению кинетической энергии шины, эквивалентен:

    \ begin {align} W_ {KE} & = W_ {Simon} — W_f \\ & = 5873 — 5500 \\ & = 373 \ J \ end {align}

    Шаг 3: Рассчитайте скорость шины, изменив формулу для кинетической энергии.{-1} (2 н.ф.)

    Вернуться к вопросу 7.

    Решение вопроса 8

    Изменение импульса и, следовательно, импульса фиксировано.

    I = \ vec {F} \ Delta t, поэтому \ vec {F} = \ dfrac {I} {\ Delta t}, что является обратной зависимостью.

    Единственный график, показывающий обратную зависимость, — это график C.

    Ответ: (C)

    Вернуться к вопросу 8.

    Решение вопроса 9

    Часть (a): По закону сохранения энергии, полная энергия шара фиксирована:

    KE_ {начальная} + PE_ {начальная} = KE_ {конечная} + PE_ {конечная}

    Шаг 1: Рассчитайте начальную кинетическую энергию мяча.2) = 1 \ J

    Шаг 2: Рассчитайте конечную кинетическую энергию шара.

    Когда мяч достигает земли, вся потенциальная энергия мяча преобразуется в кинетическую, поэтому PE_ {final} = 0.

    Отсюда:

    \ begin {align} KE_ {final} & = KE_ {initial} + PE_ {initial} \\ & = 1 + mgh \\ & = 1 + (0.5) (9.8) (10) \\ & = 50 \ J \ end {align}

    Шаг 3: Найдите выражение для количества движения только в терминах кинетической энергии и массы.

    Из формулы для кинетической энергии v = \ sqrt {\ dfrac {2KE} {m}}.

    Мы можем подставить это в формулу для импульса p = mv, чтобы получить новое выражение в терминах KE и m:

    \ begin {align} p & = mv \\ & = m \ sqrt {\ dfrac {2KE} {m}} \\ & = \ sqrt {2mKE} \ end {align}

    Шаг 4: Подставьте вычисленные значения в это уравнение, чтобы вычислить импульс мяча.

    Подстановка вычисленного значения для KE_ {final} дает:

    \ begin {align} p & = \ sqrt {2mKE} \\ & = \ sqrt {2 \ times 0.{-1}

    Часть (b)

    Шаг 1: Переформулируйте первое уравнение движения, чтобы получить выражение, связывающее время и ускорение.

    Преобразуя первое уравнение движения, мы получаем выражение, связывающее известные переменные время (t) и ускорение (a):

    \ begin {align} v & = u + at \\ vu & = at \\ & = 6 \ times 3 \\ \ поэтому v — u & = 18 \ end {выровнено}

    Шаг 2: Используйте это выражение для вычисления импульса. 2} \\ & = 55461.{-1} \ N25.6 \ градус E \ end {align}

    Шаг 2: Используйте значение начального импульса, вычисленное на шаге 1, чтобы рассчитать окончательную скорость автомобиля и грузовика вместе.

    Закон сохранения импульса гласит, что «сумма импульсов до столкновения равна сумме импульсов после столкновения». Следовательно, начальный импульс равен конечному импульсу. Используя это, мы можем вычислить окончательную скорость автомобиля и грузовика:

    \ begin {align} v & = \ frac {p} {m} \\ & = \ frac {55462} {4000} \\ & = 13 .{-1} N25.6 \ степень E

    Вернуться к вопросу 10.

    Нужны дополнительные практические вопросы по динамике модуля 2?

    Проверьте свое понимание Модуля 2 «Динамика» всего за 10 минут с помощью настраиваемого конструктора викторин Learnable. Для каждого модуля есть более 500+ вопросов в стиле экзамена. Мгновенная обратная связь позволяет мгновенно исправить ваши заблуждения. Попробуйте Learnable бесплатно прямо сейчас.

    Автор DJ Kim

    DJ является основателем Learnable и страстно интересуется образованием и технологиями.Он также является автором ресурсов по физике на Learnable.

    Learnable Education и www.learnable.education, 2019. Несанкционированное использование и / или копирование этого материала без явного и письменного разрешения автора и / или владельца этого сайта строго запрещено. Выдержки и ссылки могут быть использованы при условии, что полная и четкая заслуга дана обучаемому образованию и www.learnable.education с соответствующим и конкретным указанием на исходное содержание.

    Удивительная простота в моделировании динамического гранулированного вторжения

    ВВЕДЕНИЕ

    Вторжение в сухую гранулированную среду (GM) может создавать сложные реакции потока и силы, где среда может проявлять как твердые, так и жидкие характеристики.GM упруго деформируется под напряжением, как твердое тело, но начинает течь как жидкость, как только выполняется критерий текучести. Большие вариации напряжения, импульса и объемной доли ГМ в разных областях часто приводят к сложной динамике системы, демонстрирующей многофазные характеристики ( 1 , 2 ). Сложность потока также делает интерпретацию сил сопротивления нетривиальной, если нарушитель повторно взаимодействует с деформированной областью ( 3 , 4 ), поскольку GM теперь имеет новое неоднородное состояние вблизи поверхности.Связанную систему злоумышленника и СМИ становится сложно моделировать; Неоднородный поток и многофазность среды часто ограничивают моделирование методами дискретных частиц, которые отслеживают отдельные зерна, в отличие от жидкостей, которые можно решить с помощью уравнений Навье-Стокса.

    Обычное гранулированное вторжение включает твердое или гибкое твердое тело, проникающее в GM и использующее силу сопротивления, чтобы привести себя в состояние передвижения (см. Рис. 1). Если тело медленно вторгается в GM, то гранулярное напряжение возникает независимо от скорости проникновения, и сила сопротивления на вторгающемся теле остается в квазистатическом пределе ( 5 , 6 ).Однако могут возникнуть различные сценарии вторжения, которые деформируют среду достаточно быстро, что влияет на результирующую силовую реакцию и, следовательно, на поведение локомотива. Примеры таких вторжений включают приложения баллистики, удары метеоров, быстрое перемещение и многие промышленные процессы ( 1 , 7 10 ).

    Рис. 1 Примеры передвижения по зернистым поверхностям с различной скоростью.

    ( A ) Марсоход Wheel of the Curiosity (диаметр, 50 см) ( 36 ), ( B ) бегущий человек ( 37 ), ( C ) Робот RHex с C-образными ногами (Длина конечности C-образной ноги, 18 см) ( 38 ) и ( D ) гоночный мотоцикл для бездорожья (диаметр, 50 см) ( 39 ).Фотографии: (A) тепловизор MAHLI Curiosity, НАСА; (B) А. Сингх, www.pexels.com; (C) Г. К. Хейнс, А. М. Джонсон и Д. Э. Кодичек, Университет Пенсильвании; (D) Дэниел, www.pexels.com.

    Передвижение жесткого колеса является примером системы, которая объединяет эти эффекты, демонстрируя многофазное зернистое поведение, сложные взаимодействия зерна с поверхностью и повторное взаимодействие с деформированной средой. Жесткие колеса, подобные тем, которые используются в планетарных вездеходах ( 11 ), непрерывно срезают, а иногда и быстро деформируют местный GM ( 12 ), чтобы двигаться по рыхлому грунту.Эти вторжения, особенно в случаях с высокими угловыми скоростями, вызывают поведение материала подложки, существенно отклоняющееся от его квазистатического отклика, обусловленное потенциально нетривиальным взаимодействием поверхности с колесом. Таким образом, мы сначала сосредотачиваемся на движении жесткого колеса как диагностическом сценарии сложного вторжения, которое включает в себя широкий спектр нетривиальных эффектов.

    Мы предлагаем непрерывную структуру для вторжения, основанную на условии текучести при трении и свободного отделения. Мы реализуем каркас численно, используя метод материальной точки (MPM).Наш анализ проникновения начинается с сосредоточения на приводных круговых колесах с грунтозацепами — грунтозацепы представляют собой радиальные выступы конечного размера по окружности колеса, которые облегчают сцепление с дорогой. Колеса с гуськом обычно используются для передвижения по мягкому грунту ( 11 15 ). Наряду со сценариями медленного и быстрого передвижения на колесах моделируются два дополнительных семейства тестовых примеров, погружение боковой пластины и «четырехлопастные бегуны», которые сравниваются с известными в литературе результатами для проверки способности модели фиксировать динамику сложных гранулярных вторжений. .Предлагаемая нами модель континуума отражает нетривиальные зависящие от скорости явления, проявляющиеся в сложных интрузиях, хотя ее основные уравнения не зависят от скорости. Наша работа показывает, как единая континуальная интерпретация GM может представлять несколько сценариев вторжения, неявно согласовывая различные инерционные эффекты.

    Рис. 2 Аппарат для экспериментов с жесткими колесами и моделирования сплошных сред.

    ( A ) CAD модель экспериментальной установки. ( B ) Пример экспериментальных данных временных рядов для скорости перемещения (вверху) и погружения (внизу) при низких значениях ω (20 об / мин, сплошные линии) и высоких ω (50 об / мин, пунктирные линии) соответственно. t avg∣ s и t avg∣ l показывают временные окна, используемые для усреднения данных с низким и высоким ω, соответственно. ( C ) Схематическое представление явного интегрирования по времени на этапе MPM, при котором фоновая сетка помогает интегрировать движение на множестве материальных точек континуума. Сплошные круги (красные) — материальные точки (лагранжевые трассеры), а квадраты (синие) — узлы фоновой сетки. ( D ) Моделирование континуума образца с использованием MPM.Построенное поле представляет собой эквивалентную скорость пластической деформации.

    Мы также получаем глобальное физическое понимание динамики вторжений, анализируя решения по пластичности, которые направляют разработку модели пониженного порядка для вторжений, которую мы называем теорией динамической силы сопротивления (DRFT). Мы показываем, что DRFT точно моделирует все рассматриваемые случаи частичного вторжения. Объединив существующую литературу, моделирование континуума и экспериментальную проверку, мы идентифицируем соответствующую физику, которая входит в DRFT, и ее интерпретацию как поправки к существующей модели квазистатической теории сопротивления (RFT) ( 14 , 16 , 17 ) для медленного вторжения.Выявлены ключевые эффекты, которые вызывают поведение, зависящее от скорости, и после включения DRFT позволяет быстро рассчитать ожидаемые силы сопротивления в GM.

    РЕЗУЛЬТАТЫ И ОБСУЖДЕНИЕ

    Эксперименты по перемещению колес

    На рисунке 2A показана модель автоматизированного проектирования (CAD) лабораторной установки, используемой для проведения экспериментов по перемещению колес в этом исследовании, а на рис. 2B показана наша методология сбора данных. Более подробная информация об экспериментальной установке представлена ​​в материалах и методах (и в фильме S3).

    На рис. 3 (A и B) показаны тенденции установившейся скорости поступательного движения и опускания (соответственно) с увеличением угловой скорости для свободного передвижения колеса с куропаткой. Эксперименты указывают на появление темпозависимого эффекта при передвижении колес; Увеличение проскальзывания, сопровождающееся увеличением опускания колес, нарушает линейный тренд скорости в зависимости от ω, наблюдаемый в квазистатической области ω <30 оборотов в минуту (об / мин) (соответствует ω / ω o <0.46 на рис. 3А).

    Рис. 3 Сравнение экспериментов по перемещению колес и моделирования континуума.

    Изменение скорости поступательного движения ( A ) и опускания ( B ) из экспериментов (синий) и решений моделирования континуума (черный). Результаты обезразмерены с использованием характеристической скорости системы ( г ℓ) 0,5 (= 1440 мм / с) для скорости поступательного перемещения; характерная длина системы ℓ (= 212 мм) для отвода воды; и характеристическая угловая скорость ω o (= ( g / ℓ) 0.5 = 65 об / мин) для угловой скорости, где g представляет силу тяжести, а l представляет собой внешний диаметр колеса. ( C ) Скорости полей гранулированного потока, полученные в результате моделирования и экспериментов в континууме (PIV) для медленного (30 об / мин, вверху) и быстрого (60 об / мин, внизу) движения колеса. Данные усредняются по эффективному вращению на 0,1 рад (для PIV), при этом оранжевые области представляют собой среднее положение колеса. Смотрите фильмы с S4 по S6 для получения более подробной информации. Размеры колес приведены в таблице 1.( D ) Графики, показывающие изменение скорости зерна из моделирования континуума и экспериментов PIV вдоль радиального направления непосредственно под центром колеса; Обратите внимание, что некоторое трение стенки пластиной из оргстекла существует в эксперименте, но не в растворе континуума. Ключевые структурные особенности потока под колесом согласуются между экспериментами и моделью в (C) и (D).

    Анализ гранулярного вторжения с помощью моделирования континуума

    Мы используем простую гранулярную модель континуума, которая отражала динамику нарушителя в предыдущих исследованиях в медленных, квазистатических режимах ( 14 , 18 ).Семена мака (PS), модель GM, используемая в этом исследовании, моделируются как гранулированный континуум с критерием текучести Друкера-Прагера (на основе трения, не зависящего от скорости), несжимаемым пластическим сдвигом и критерием разделения материала на среда без напряжений, когда плотность ниже критической. Эта реология может быть определена двумя одновременными ограничениями, показанными ниже, описывающими поведение материала при разделении и условие текучести при сдвиге Свободное разделение: (ρ − ρc) P = 0 и P≥0 и ρ≤ρc (1) Податливость при трении: γ · (τ− μsP) = 0 и γ · ≥0 и τ≤μsP (2) для i , j = 1,2,3.Определим σij ′ = σij + Pδij как девиаторную часть тензора напряжений Коши, P = — σ ii /3 как гидростатическое давление, τ = σij′σij ′ / 2 как эквивалентное напряжение сдвига, μ s как коэффициент объемного трения и ρ c как критическая плотность плотноупакованных гранул. Тензор (пластической) скорости потока равен D ij = (∂ i v j + ∂ j v i ) / 2, а γ · = 2DijDij — эквивалентная скорость сдвига. .При пластическом сдвиге предполагается, что напряжение и скорость потока совпадают (например, σij ′ / 2τ = Dij / γ ·). Модель развивает поток, решая уравнения баланса импульса, ∂jσij + ρgi = ρv · i. Ниже критерия текучести зерна действуют как линейно-упругое твердое тело, так что наша модель является упругопластической в ​​плотном режиме. Мы предполагаем, что постоянный коэффициент поверхностного трения описывает взаимодействие гранулированного континуума с поверхностями твердых тел. Плотность среды, внутреннее трение материала, трение среда-поверхность и другие входные данные материала включены в таблицу S1 (в дополнительных материалах).

    Мы используем алгоритм MPM, описанный у Дунатунги и Камрина ( 19 , 20 ), чтобы реализовать эти определяющие уравнения, предполагающие двумерное (2D) движение с плоской деформацией. Схематическое представление явного шага MPM интегрирования по времени показано на фиг. 2C; материальные точки несут данные континуума и перемещаются на каждом шаге с помощью фоновой сетки (подробнее см. в материалах и методах). На рисунке 2D показан образец движения колеса с использованием MPM, отображающий изменение эквивалентной скорости пластического сдвига в системе.

    Тенденции установившейся скорости поступательного движения и опускания при изменении ω, полученные с помощью моделирования континуума, показаны на рис. 3 (A и B). Моделирование континуума успешно отражает экспериментальные тенденции движения колес; в частности, модель фиксирует плато на нормализованной кривой v — ω при правильной скорости вращения и правильно предсказывает увеличенное опускание с увеличением скорости вращения.

    Чтобы проверить надежность результатов, мы также применили небольшие изменения к начальному состоянию экспериментальной и смоделированной систем, включая изменения начальной глубины колеса, начальной скорости колеса и скорости разгона колеса, и наблюдали, что установившееся состояние результаты были нечувствительны к этим вариациям ( 21 ).

    Для дальнейшей проверки прогнозов модели мы провели эксперименты по визуализации полей подземных потоков и сравнили их с моделью. В экспериментах колесо помещается рядом с прозрачной пластиной из плексигласа, чтобы камера могла зафиксировать движение зерна под ним с помощью велосиметрии изображения частиц (PIV) ( 6 ). Поля скоростей в зернах для случаев 30 и 60 об / мин из моделирования континуума и экспериментального PIV-анализа показаны на рис. 3 (C и D). Мы полагаем, что сопротивление стенок пластиной из плексигласа, вероятно, приводит к тому, что поток гранул в эксперименте в целом будет медленнее, чем в модели; однако ключевые структурные особенности потока под колесом согласуются между экспериментом и моделью.Оба показывают зону материала перед колесом, выталкиваемого вперед, и широкую зону под колесом и позади него, выталкиваемую назад. Зона обратного потока также увеличивается с увеличением ω из-за более высокого увлечения потока и движения материала при более высоких ω.

    К моделям сокращенного порядка

    Основным преимуществом в определении точной модели континуума для системы является возможность ее использования для извлечения глобальных упрощений динамики системы, которые можно использовать для разработки моделей с дальнейшим сокращением.Например, в предыдущей работе о медленном квазистатическом вторжении Аскари и Камрин ( 18 ) обнаружили связь между фрикционной податливостью и моделью силы вторжения пониженного порядка, называемой гранулярной RFT ( 16 ). Успех существующей континуальной модели медленного и быстрого передвижения в колесах (и других сценариев вторжений в этом исследовании) побуждает нас задаться вопросом, существует ли RFT-подобная модель пониженного порядка для сложных, быстрых вторжений и может ли она быть выведена на основе о явлениях, наблюдаемых в рамках модели континуума.Мы начнем с определения квазистатической формы RFT и оценки ее прогнозов для динамики передвижения колес.

    RFT — это эмпирическая методология, которая оказалась успешной в оценке силового отклика для произвольной формы вторгающейся геометрии в квазистатическом пределе, позволяя прямое моделирование движения в гранулированных объемах ( 14 , 16 , 17 ). RFT предполагает, что напряжение на небольшом поверхностном элементе злоумышленника следует локализованной формуле, в которой зависит только от движения, местоположения и ориентации этого элемента ( 22 ).Эта локальная формула разделяет реакцию на напряжение между элементами поверхности злоумышленника, тем самым позволяя RFT прогнозировать силы вторжения с помощью численных расчетов почти в реальном времени.

    В системе координат, где z указывает положительно вверх с гранулированной свободной поверхностью на z = 0, а x является выбранной горизонтальной осью, перпендикулярной z , RFT предполагает вектор силы, перпендикулярной площади (или тяги) t , на каждом элементе поверхности можно записать как t = α (β, γ) H (- z ) ∣ z ∣, в зависимости от угла ориентации элемента (β) , угол скорости (γ) и вертикальная глубина от свободной поверхности (∣ z ∣), где H является функцией Хевисайда.Эмпирический вектор тяги на глубину α (β, γ) = (α x (β, γ), α z (β, γ)) измеряется с помощью экспериментов по вторжению небольших плит, которые меняют β и γ. Суммируя эти локально определенные силы сцепления, RFT прогнозирует чистую силу сопротивления и момент на всей поверхности нарушителя S . Например, RFT дает следующую формулу силы вторжения: F = ∫Sα (β, γ) H (−z) ∣z∣ dA (3)

    На рисунке 4 (A и B) показаны результаты применения квазистатического RFT (сплошной синяя линия на рис.4, А и Б) при моделировании передвижения колёсных рябчиков. При реализации модели передвижения RFT мы также используем «гипотезу переднего края», чтобы гарантировать, что силы сопротивления, испытываемые колесом, состоят только из элементов поверхности, которые движутся «в» песок, т. Е. Поверхностей, внешняя нормаль которых ( n ) и скорость ( v ) образуют положительный внутренний продукт ( n · v > 0). Мы используем установленные функции RFT α x и α z , для GM, используемой в наших экспериментах ( 16 ).На рисунке 4B показано, что хотя RFT фиксирует тенденции изменения скорости в зависимости от ω при низких значениях ω, при более высоких ω он не прогнозирует кинематику движения колеса. RFT предсказывает линейную зависимость между угловой и поступательной скоростями, которая соответствует динамике экспериментов на низких скоростях, но расходится с увеличением ω. Тот факт, что квазистатический RFT предсказывает, что установившаяся скорость круглого колеса всегда будет постоянной кратной вращению колеса, может быть показан как следствие независимости скорости отношения тяги RFT в формуле.3 (подробности см. В разделе S2).

    Рис. 4 Эксперименты в сравнении с RFT.

    ( A ) Снимок квазистатического моделирования RFT, использованный для изучения передвижения колёсных рябчиков. Направление и величина (нормированная) скорости и сопротивления показаны красными и синими стрелками, соответственно, вдоль элементов поверхности границы колеса. ( B ) Скорость поступательного движения (вверху) и поведение колеса при опускании (внизу); экспериментальное среднее и стандартное отклонение 1σ (голубые данные) и результаты RFT с локальной модификацией λρ v 2 (сплошные линии).Результаты в (B) безразмерны, как показано на рис. 3. Направление увеличения λ указано (λ = 0, 1, 25, 50 и 100). Красные сплошные линии с λ = 0 [в (B)] соответствуют квазистатическим результатам RFT.

    Использование континуума для физического понимания

    Важным шагом в разработке общей модели пониженного порядка для сценариев высокоскоростного гранулярного вторжения является определение основных физических факторов. При гранулярных вторжениях эффекты скорости могут возникать по разным физическим причинам.Повышенные вибрации в среде могут привести к псевдоожижению материала на высоких скоростях и снижению его прочности ( 23 ). Увеличение скорости могло бы уменьшить трение на границе раздела колесо / среда (на динамическое падение трения), что, в свою очередь, могло бы уменьшить тягу на колесах. Быстрые потоки также могут иметь значительную микроинерцию, которая делает скорость реологии зависимой, заставляя отношение напряжений μ ≡ τ / P зависеть от скорости сдвига через «инерционное число» I , где I = γ · d2ρs / P, где γ · — скорость сдвига, d — средний диаметр зерна, ρ s — плотность твердых частиц и P — местное давление ( 5 ).Более того, обычная макроинерция (то есть член ρv · i в уравнении баланса импульса) добавляет инерционные телесные силы, которые могут изменить поток среды и ее сопротивление нарушителю.

    Предсказать доминирующий эффект (эффекты) скорости трудно, используя только эксперименты. В этом отношении наш подход к моделированию континуума в значительной степени помогает исключить кандидатов из возможных эффектов скорости, указанных выше. Ключевым моментом является напоминание о том, что наша модель реализует нечувствительное к скорости фрикционное взаимодействие поверхности без динамического падения трения на границе раздела колесо-песок и не зависящую от скорости конститутивную модель без зависимости от инерционного числа и без учета термализации или псевдоожижения материала.Однако модель включает макроинерцию в уравнения баланса импульса. Тот факт, что модель континуума успешно фиксирует динамику колеса вместе со многими другими сценариями гранулярного вторжения (обсуждаемыми позже), указывает на то, что наблюдаемые эффекты скорости должны быть согласованы исключительно с макроинерцией (ρv · i). В то же время глобальные последствия локальных макроинерционных сил могут быть незаметными и зависеть от конкретной системы и ее динамики.

    На основе этого понимания, наряду с анализом континуальных решений для передвижения колес и других сценариев детализированного вторжения из литературы, мы теперь предлагаем и тестируем более общий RFT, который охватывает область от медленных до быстрых вторжений в GM, которые мы называем DRFT.

    Теория динамической силы сопротивления

    DRFT изменяет квазистатический RFT двумя способами для учета макроинерционных эффектов. Во-первых, мы добавляем вклад потока импульса, который мы называем динамической инерционной поправкой. Этот член необходим для передачи импульса сыпучему материалу, окружающему нарушителя. Этот термин также согласуется со многими предыдущими исследованиями высокоскоростных гранулярных вторжений ( 24 30 ) и принимает форму дополнительной зависящей от скорости силы, выражающейся в квадрате скорости.Вторая модификация, которую мы покажем критически важной для более сложных интрузий, описывает способ, которым увеличенная объемная инерция может изменить геометрию свободной поверхности. Затем изменение геометрии свободной поверхности влияет на силы сопротивления через зависимость RFT от глубины. Обозначим эту модификацию как динамическую структурную коррекцию. Вместе DRFT вводит следующую формулу для сцепления с элементом поверхности t = α (β, γ) H (−z˜) ∣z˜∣ − nλρvn2 (4) где ∣z∼∣ указывает эффективную глубину элемента поверхности.То есть z∼ = z + δh, где δ h представляет уменьшение высоты свободной поверхности в зоне, влияющей на тягу в ( x , z ). Вспомните, что n представляет собой внешнюю нормаль к элементу поверхности (и — n внутрь), и мы определяем v n как нормальный компонент поверхностной скорости. Чтобы использовать DRFT, необходимо определить соответствующее значение δ h для каждого элемента поверхности нарушителя как функцию движения нарушителя и соответствующее значение λ, скалярную подгоночную константу O (1).Подобно RFT, DRFT утверждает локализованную формулу для расчета напряжений на недрах нарушителя и, таким образом, позволяет моделировать движение нарушителя почти в реальном времени.

    Понимание динамической инерционной коррекции

    Мы уделим время обсуждению двух динамических коррекций, включенных в DRFT, начиная с динамической инерционной коррекции. Анализ уравнений баланса количества движения при определенных упрощающих обстоятельствах (см. Раздел S1) позволяет сделать вывод, что переход от квазистатического потока к более быстрому потоку происходит с увеличением силы сопротивления как ρAvn2, аналогично динамическому давлению в жидкости, где A — зона злоумышленников.Физически этот термин представляет собой силу реакции, возникающую при передаче импульса GM.

    Ряд предыдущих исследований ( 24 30 ) моделировали зависимость силы вторжения от скорости аналогичным образом, добавляя член, пропорциональный квадрату нормальной скорости, к «статическому» члену, зависящему от глубины. Изучение экспериментальных данных в ( 26 , 31 ) согласуется с зависящим от скорости сложением силы формы λρAvn2 в простых вертикальных и горизонтальных интрузиях (см. Рис.S2 и S3 и фильмы S1 и S2), где λ — скалярная константа аппроксимации O (1), которая учитывает определенные приближения в анализе (см. Раздел S1).

    Естественно спросить, достаточно ли одного добавления квадрата скорости к квазистатическому соотношению RFT для объяснения зависимости скорости, наблюдаемой в общих сценариях вторжения, включая передвижение на колесах. Мы предполагаем, что поверхностное сцепление моделируется так, чтобы подчиняться соотношению в формуле. 5 ниже и используйте это соотношение для повторной оценки задачи передвижения на колесах t = α (β, γ) H (−z) ∣z∣ − nλρvn2 (5)

    На рисунке 4B показаны результаты для различных значений λ.Случай λ = 0 представляет ранее обсужденный квазистатический RFT на этих графиках. Введение члена силы инерции (λ> 0) добавляет новый вклад силы, имеющий компоненты результирующей силы, направленные вверх и противоположные горизонтальному направлению поступательного движения колеса. Эта направленная вверх сила приводит к уменьшению оседания колеса, в отличие от экспериментального наблюдения. Величина этих дополнительных сил очень мала; предварительный коэффициент λ был изменен от 1 до 100 в попытке согласовать эксперименты, но это мало влияет на результат, и тенденции как скорости, так и погружения не могут быть согласованы (рис.4Б). Ясно, что одной только динамической инерционной коррекции недостаточно для описания этого набора тестов.

    Понимание динамической структурной коррекции

    Чтобы понять обоснование динамической структурной коррекции в DRFT, мы начнем с рассмотрения пространственного изменения величин скорости пластической деформации из моделирования континуума для случаев с низким и высоким ω, показанных на рис. 5D. Графики позволяют визуализировать, как разные части колеса получают свои силы сопротивления от разных зон GM.Хотя профили скорости деформации меняются с увеличением угловой скорости, основные модели сдвига остаются аналогичными. Срезанный материал достигает свободной поверхности гранулированного объема в двух зонах. Примерно половина потока, исходящего от передней кромки колеса, достигает свободной поверхности задней задней поверхности колеса. Остальные линии тока проходят до свободной поверхности передней передней поверхности колеса. Высота свободной поверхности на задней стороне колеса уменьшается с увеличением ω; качественно по мере роста ω колесо выталкивает материал с тыльной стороны.Уменьшение высоты свободной задней поверхности предполагает уменьшение напора и, как следствие, ослабление материала в задней зоне сдвига. Это ключевое наблюдение, которое мотивирует форму динамической структурной коррекции.

    Рис. 5 Динамический RFT фиксирует эксперименты и моделирование континуума.

    Изменение скорости поступательного движения колеса ( A ) и осадки ( B ) по результатам экспериментов по сравнению с квазистатическими RFT, DRFT и DRFT без какой-либо динамической структурной коррекции (т.е., имея только поправку ∼ρ v 2 ). ( C ) Предполагаемые зоны влияния и эффективное изменение свободной поверхности для построения динамической структурной коррекции; δ h представляет собой промежуток между эффективными положениями передней и задней свободной поверхности. Данные MPM (красные кружки) и эмпирическая аппроксимация (синяя пунктирная линия) для δ h . ( D ) Изменение эквивалентной величины скорости пластической деформации, полученной с использованием моделирования континуума MPM для медленного (30 об / мин) и высокоскоростного (90 об / мин) движения колеса.См. Фильм S7 для визуализации изменений во времени. Результаты (A) и (B) безразмерны, как показано на рис. 3.

    На рис. 5C показано уменьшение высоты свободной поверхности δ h , измеренное на основе моделирования континуальной модели путем определения самой нижней точки, образующей заднюю часть. контакт с колесом, для которого гидростатическое давление → 0. Чем быстрее вращается колесо, тем глубже опускается эта точка. Учитывая небольшое количество параметров в модели континуума, анализ размеров полезен; для данного материала подложки предлагается форма δ h = r · ψ ( r ω 2 / g ) для некоторой функции ψ.Неожиданно оказалось, что ψ хорошо аппроксимируется тождественной функцией. Подгонка δ h = r ( r ω 2 / g ) и результаты моделирования континуума на фиг. 5C показывают хорошее согласие. В сочетании с пониманием, развитым в предыдущем разделе, форма эффективной свободной поверхности аппроксимируется с помощью простой перегородки, как показано на рис. 5C, с задней зоной колесной пары, имеющей постоянное уменьшение высоты свободной поверхности h назад , отличающаяся от начальной высоты свободной поверхности (невозмущенная средняя высота) на член δ h = r ( r ω 2 / g ).Чтобы выбрать разделительный угол, разграничивающий переднюю и заднюю зоны потока, мы решили поровну разделить зону контакта для ведомых колес. Наш выбор обусловлен простотой этого разделения, также соблюдается аналогичное разделение зон контакта для представления тяги на колесах Хэмблтона и Дрешера ( 32 ). Эта новая модель изменяет эффективную высоту свободной поверхности только для элементов поверхности, расположенных ближе к задней части выступающей поверхности колеса.

    Включив эту формулировку эффективной высоты свободной поверхности, мы теперь приходим к DRFT, уравнение.4. Мы реализуем эту модель DRFT, используя ту же структуру неявного кода RFT, описанную в разделе «Материалы и методы», используя λ = 1 и ρ ≈ ρ c = 638 кг / м 3 . Тенденции скорости поступательного движения и погружения относительно ω теперь показывают хорошее согласие между экспериментом и DRFT (рис. 5, A и B). Мы также включаем для сравнения, каково решение, когда используется только динамическая инерционная коррекция. Хотя DRFT сочетает в себе обе динамические коррекции, очевидно, что динамическая структурная коррекция преобладает над динамической инерционной коррекцией в случае передвижения на колесе.Хотя для простоты мы предположили, что разделение между двумя контактными зонами происходит на полпути через границу раздела колеса и песка, это можно увидеть на рис. 3C и 5D видно, что перегородка может быть ближе к передней части колеса. Это могло объяснить наше небольшое завышение скорости при высоких значениях ω (рис. 5A). Второй набор испытаний колес с грунтозацепом с участием колеса меньшего размера включен в Дополнительные материалы (см. Рис. S1 и раздел S3), и DRFT работает одинаково хорошо без необходимости переоборудовать функцию для ψ, используемую для δ h .

    Соглашение с DRFT предполагает, что переход от низкого к высокому скольжению при передвижении на колесах (где скольжение = 1 — v / r ω для v — это поступательная скорость, r — номинальный радиус, и ω — угловая скорость колеса) происходит в основном из-за того, что более быстро вращающиеся колеса удаляют материал из-за колеса, что снижает давление в задней зоне, тем самым ослабляя основу материала, которая в противном случае обеспечивала бы подмости, на которых колесо толкает.При обновлении RFT с учетом этого эффекта была должным образом отражена динамика сложного сценария движения колеса в структуре моделирования пониженного порядка.

    Дополнительные проверочные исследования для модели континуума и DRFT

    Испытания колес обеспечивают сложный сценарий вторжения и имеют динамическую структурную коррекцию, которая намного больше, чем инерционная коррекция. Чтобы проверить надежность нашего подхода к моделированию континуума и уравнения. 4 для DRFT, теперь мы исследуем обратную ситуацию с двумя дополнительными наборами моделирования — злоумышленников с погруженными пластинами и движущихся бегунов.Мы оцениваем эти случаи на основе данных из непрерывных решений, проверки на соответствие литературным источникам и аргументов в предыдущем разделе и ожидаем, что динамическая структурная коррекция будет небольшой, а динамическая инерционная коррекция будет доминировать. Визуально эти случаи представляют собой два отдельных класса злоумышленников. В то время как перетаскиваемые пластины представляют собой принудительное движение, полозья представляют собой класс самодвижущихся локомоторов, которые могут показаться похожими на ранее изученные колеса. Тем не менее, в силовых реакциях в обоих случаях преобладает динамическая инерционная коррекция (более подробная информация в следующих разделах), и они не имитируют поведение колеса с тетеревами.Таким образом, эти отдельные случаи проверяют широту возможностей моделирования DRFT.

    Горизонтальное проникновение под воду . Тонкие пластины, погруженные в GM на разную фиксированную глубину (от 20 до 40 мм), перемещаются по горизонтали с разной скоростью с использованием моделирования континуума. Модель континуума работает при плоской деформации, где длина пластины составляет 0,016 м, а эффективная плотность среды составляет 900 кг / м 2 . Выбранная плотность аналогична измельченному углю или мрамору. Заштрихованные кружки на рис.6A показаны наблюдаемые изменения силы сопротивления в зависимости от скорости сопротивления. Экспериментальные исследования Schiebel et al. ( 31 ) обнаружил, что изменение сил сопротивления в таком сценарии следует тенденции K z ∣ + λρ Av 2 (см. Рис. S1), где K и λ — постоянные, z ∣ — глубина пластины ниже свободной поверхности, ρ — эффективная плотность гранул, A, — площадь пластины, и v — горизонтальная скорость пластины.Наше моделирование континуума также демонстрирует ту же тенденцию (рис. 6A). В самых медленных случаях ( v ∼ 0) мы получаем зависимость линейной силы от глубины, F сопротивление = K z ∣ для K = 580 Н / м. По мере увеличения скорости мы обнаруживаем, что предсказания континуума хорошо совпадают с данными на трех разных глубинах для λ = 1,1. Между прочим, такое же значение λ также соответствует зависимости от скорости, наблюдаемой в Schiebel et al. ( 31 ) экспериментов для горизонтально перемещаемых злоумышленников на свободной поверхности.

    Рис. 6 Моделирование медленного и быстрого внедрения плит.

    ( A ) Данные MPM континуума (цветные кружки) и K z ∣ + λρ Av 2 подходит (пунктирные линии) для горизонтальных вторжений на различной глубине (∣ z ∣) ([[ 20,30,40] мм), где K = 580 Н / м и λ = 1,1. ( B ) Изменение эквивалентной скорости пластической деформации для (вверху) низкоскоростного (0,04 м / с) и (внизу) высокоскоростного (0,64 м / с) случаев проникновения (на глубине 30 мм).Смотрите фильм S2 для видео. Моделирование плоской деформации.

    Полное понимание результирующей формы трендов силы сопротивления может быть получено путем наблюдения за результатами моделирования континуума в контексте DRFT. На рис. 6В показаны профили деформации вокруг пластины при двух выбранных скоростях (которые различаются примерно на порядок). Профили на рис. 6В высокоскоростного и низкоскоростного проникновения предполагают, что силы проникновения злоумышленника возникают из-за прижатия гранулированного материала перед пластиной вверх и вправо к общей высоте свободной поверхности, h спереди .Зона обратного потока, которая изменяется при медленном проникновении по сравнению с высокоскоростным, либо находится в отдельной фазе, либо вновь консолидируется по мере того, как опускается и заполняет зазор за движущейся пластиной. Точно так же задняя среда вносит незначительный вклад в силу резистивной пластины; никакая часть задней поверхности пластины не является «передней кромкой», удовлетворяющей требованиям n · v > 0, поэтому силы там примерно исчезают. Это контрастирует с корпусом колеса с гусиным грунтом, где из-за вращения значительная часть задней половины колеса является передней кромкой, которая может нетривиально взаимодействовать со средой за колесом.Таким образом, мы ожидаем незначительной динамической структурной коррекции горизонтального сопротивления пластины из-за отсутствия передней кромки на задней поверхности пластины и приблизительно независимой от скорости h передней . Действительно, соотношение сил F сопротивления = K z ∣ + λρ Av 2 , полученное в результате экспериментов, а также моделирования континуума, отображает только динамическую инерционную коррекцию DRFT, как и ожидалось. Эти результаты согласуются с нашей гипотезой и подтверждают прогноз DRFT для подводного горизонтального вторжения.По тем же причинам, которые только что обсуждались, мы ожидаем, что симметричное вертикальное вторжение плит также вызовет незначительную структурную коррекцию; см. дополнительные материалы (рис. S3) для получения подробной информации и подтверждения против DRFT. Обратите внимание, что в наших исследованиях сопротивления плит мы ограничили нашу глубину проникновения в пределах O (1) фактора ширины плиты. Этот диапазон глубин указывает приблизительные пределы RFT, поскольку за пределами таких глубин предположения RFT (такие как линейная зависимость гранулярного сопротивления от глубины) начинают ухудшаться ( 33 ).

    Направляющая с четырьмя лопастями . В то время как перетаскиваемые пластины вынуждены двигаться с заданной скоростью, мы также изучаем самодвижущийся локомотор, бегун с четырьмя лоскутами, скорость передвижения которого определяется посредством взаимодействия самодействующих конечностей локомотора (движение лоскута) и динамики субстрата. (геометрические данные приведены в таблице S1). Небольшое количество закрылков, наряду с большим отношением длины закрылка к внутреннему радиусу, сводит к минимуму взаимодействие между соседними проникновениями закрылков в результирующий гранулированный поток рабочего колеса.

    Бегун черпает вдохновение в экспериментах Ли и др. ( 16 ) и Zhang et al. ( 34 ) с работающими C-образными роботами (аналогично рис. 1C). Li et al. ( 16 ) управлял своими роботами с безразмерным коэффициентом вращения (ω / ω o ) в диапазоне от 0 до 1,25 ((ω max , ω o ) = (240,190) об / мин) и наблюдал уменьшение скольжения с увеличивая угловую скорость в своих экспериментах. Аналогичным образом Zhang et al. ( 34 ) проверил движение в большем диапазоне ω / ω o от 0 до 3,8 ((ω max , ω o ) = (720,190) об / мин) и заметил, что в более высоком диапазоне спинов, опускание в их экспериментах отрывается от тенденций, наблюдаемых Li et al. ( 16 ), то есть роботы поднимаются выше своей глубины покоя. Их бегущие роботы демонстрируют качественно противоположное поведение по сравнению с гусевыми колесами: по мере увеличения скорости вращения бегуны меньше тонут и двигаются быстрее, тогда как колеса опускаются больше и движутся медленнее.Мы исследуем, заложена ли уже фундаментальная физика такого качественно обратного поведения в наше моделирование континуума и, как следствие, структуру DRFT. Поскольку наши текущие возможности моделирования континуума были ограничены задачами с натяжением на плоскости (2D), мы не можем реализовать полностью C-образного робота, работающего в 3D. Мы рассматриваем бегуна с четырьмя клапанами как представителя семейства бегунов и исследуем возможности нашей двухмерной модели континуума в моделировании такого поведения.

    При континуальном моделировании безразмерное отношение масс рабочего колеса, равное м / ρ c 𝓁 2 W для W , внеплоскостная ширина, устанавливается равной тот же диапазон (≈6), что и у соответствующих 20 грунтовых колес, показанных ранее, чтобы сравнение между бегунами и грунтованными колесами было актуальным.По тем же причинам мы сохраняем диаметр полозья, аналогичный диаметру колеса для гусей (190 мм против 212 мм). Угловая скорость бегуна изменяется в диапазоне от 10 до 300 об / мин, что соответствует диапазону безразмерного отношения вращения, изменяющемуся от ω / ω o = 0 до 4,5 (ω o = 65 об / мин). Результаты континуума (см. Рис. 7, B и C) показывают качественное согласие с выводами Li et al. ( 16 ) и Zhang et al. ( 34 ) — при увеличении скорости вращения наблюдается уменьшение эффективного скольжения и подъем колеса над глубиной покоя.Между прочим, оборот по высоте у наших бегунов был обнаружен при коэффициенте вращения ~ 1,4, аналогичном полученному Zhang et al. ( 34 ).

    Рис. 7 Запуск на GM.

    ( A ) Решения модели континуума для эквивалентной пластической деформации при возрастающих угловых скоростях ω для движения четырехлопастного бегуна (ω o = 65 об / мин). Смотрите фильм S8. Решения континуума от MPM (черная пунктирная линия с маркерами «o») и решений DRFT (сплошные линии) для поступательной скорости ( B ) и погружения ( C ) в зависимости от угловой скорости ω при движении бегуна с четырьмя закрылками.Решения DRFT для λ = 0, 2 и 4 изображены. Результаты в (B) и (C) безразмерны, как показано на фиг. 3, с ℓ = 190 мм (внешний диаметр рабочего колеса).

    Теперь мы используем модель континуума в качестве основы для оценки эффективности DRFT для бегунов. На рисунке 7A показано изменение эквивалентной пластической деформации для четырех различных угловых скоростей в модели континуума. Как и ожидалось, из-за относительно большого расстояния между вторгающимися ветвями отсутствует видимое самовзаимодействие гранулированного материала между вторжениями, и высота свободной поверхности непосредственно за вторгающимися ветвями остается неизменной, что предполагает минимальную роль динамической структурной коррекции.Это наблюдение помогает нам смоделировать эти сценарии с использованием DRFT с типичными значениями O (1), λ (λ = 0,2,4) и без динамической структурной коррекции. На рис. 7 (B и C) показаны результирующие установившиеся скорости погружения и поступательного перемещения при различных угловых скоростях из расчетов DRFT (сплошные линии). DRFT, по-видимому, улавливает кинематические тенденции эталонного решения, приближаясь к количественной точности для λ ∼ 4. С этим результатом обнадеживает тот факт, что DRFT фиксирует зависимость от ω как у полозьев, так и у гусеничных колес, которые ведут себя противоположным образом, как ω увеличивается.

    Наше исследование бегунов с четырьмя откидными створками также объясняет наблюдения вышеупомянутых исследований роботов с C-образными ногами. Мы полагаем, что квазистатическое моделирование RFT в Li et al. ( 16 ) было достаточно, потому что динамическая инерционная коррекция все еще была небольшой в их испытанном диапазоне (в нашем исследовании динамическая инерционная коррекция становится заметной только при соотношении ω / ω o , равном ∼1,2). Zhang et al. ( 34 ) переходят к более высоким скоростям вращения, выявляя нетривиальные тенденции подъема и скольжения из-за скорости, которую мы видим в решениях континуума и DRFT.

    Заключение

    В этой работе мы сосредоточились на оценке эффективности и последствий континуальной модели для проблем гранулярного вторжения вплоть до высоких скоростей, что позволяет детально моделировать сложные многофазные неоднородные гранулированные системы. Мы наблюдали два неожиданных результата. Во-первых, модель континуума, основанная только на постоянном коэффициенте трения и разделении без напряжения, может количественно моделировать сложные гранулированные вторжения в различных сценариях.Во-вторых, мы обнаружили, что всего две макроинерциальные поправки к RFT позволяют успешно моделировать гранулярные вторжения в разных скоростных режимах.

    Эти результаты получены постепенно. Путем анализа решений континуальной модели было выявлено понимание основных физических факторов, участвующих в таких сложных сценариях вторжений, что, в свою очередь, послужило стимулом для создания компонентов DRFT. DRFT обеспечивает надежное моделирование гранулярного вторжения почти в реальном времени в большом количестве случаев, включая самодвижение.Наше исследование жесткого вторжения в GM показывает, что силовая реакция на вторжение состоит из двух основных модификаций, зависящих от скорости: (i) динамическая инерционная коррекция и (ii) динамическая структурная коррекция. Динамическая инерционная поправка учитывает передачу импульса окружающему материалу, тогда как динамическая структурная поправка описывает, как быстро движущийся нарушитель может изменить напор, изменяя свободную поверхность. Оба эффекта связаны с макроинерцией среды (происходящей из члена ρv · i в уравнении с частными производными баланса импульса).Для рассматриваемых здесь сценариев микроинерционные эффекты [на реологию μ ( I )] не имеют значения, даже если движение кажется «быстрым» — предыдущие работы по быстрому проникновению снаряда ( 20 ) указывают на то, что высокие давления, которые развиваются вокруг быстрых злоумышленников, как правило, сохраняют I относительно маленьким. Таким образом, мы пришли к довольно интересному выводу, что наблюдаемая динамика, зависящая от скорости, совместима с реологией, которая не зависит от скорости. Что касается ограничений, известно, что квазистатический RFT теряет точность, когда злоумышленник находится слишком глубоко, поскольку зависимость линейной силы от глубины в конечном итоге плато в направлении подъема ( 33 ) для медленных злоумышленников.Мы ожидаем, что те же ограничения по глубине будут применяться и к DRFT.

    Dynamic RFT обладает достаточной общностью, чтобы объяснить два противоположных сценария: ослабление GM во время передвижения гусиных колес, а также усиление GM во время быстрого бега. Мы показали, что DRFT точно предсказывает поведение GM-системы в предельных случаях, то есть когда один из двух динамических эффектов является доминирующим. Необходимы дальнейшие исследования, чтобы полностью протестировать модель для смешанных случаев, когда значимы обе динамические поправки.Мы предположили аддитивность в соответствии с предыдущими представлениями о статической составляющей и инерционной составляющей силы вторжения ( 6 , 23 , 25 , 30 ). Однако не исключено, что может возникнуть более сложное функциональное сочетание.

    Хотя наше исследование в основном сосредоточено на сухом некогезивном ГМ, формулировка DRFT в гранулированных потоках предполагает существование других подобных моделей пониженного порядка для других материалов. Комбинация экспериментов и моделирования континуума оказалась жизненно важной в этом исследовании для проверки лежащих в основе физики.Предложенная структура континуума может быть легко модифицирована для охвата большого разнообразия материалов, если известны их определяющие уравнения. В будущей работе могут быть исследованы более быстрые методы прогнозирования потоков, а также различные сложные злоумышленники для систематического определения формы динамической структурной коррекции. Дальнейшие исследования могут также изучить существование аналогичных моделей пониженного порядка для связанных классов материалов, таких как GM некритического состояния, связные пески / илы и насыщенные флюидом пески.

    Благодарности: Финансирование: S.A., A.K., D.I.G. и K.K. выражаем признательность за поддержку грантов W911NF1510196 и W911NF1810118 со стороны Исследовательского бюро армии США (ARO) и поддержку Центра исследований, разработок и инженерии танковой техники армии США (TARDEC). Вклад авторов: S.A., A.K., D.G. и K.K. задумал исследование. С.А. и А.К. соавтор рукописи. S.A. выполнила численный анализ и моделирование. А.К. проводил эксперименты. Все авторы интерпретировали данные, обсуждали результаты и выдвигали теорию.К.К. и Д. руководил проектом, давал комментарии к рукописи и модифицировал рукопись. Конкурирующие интересы: Авторы заявляют, что у них нет конкурирующих интересов. Доступность данных и материалов: Все данные, необходимые для оценки выводов в статье, представлены в документе и / или дополнительных материалах. Дополнительные данные, относящиеся к этой статье, могут быть запрошены у авторов.

    Динамика и силы, викторины и тестовый пакет Уровень 11 Физика Версия № 3

    Я продаю 5 типов продуктов: очки питания, тестовые пакеты, пакеты рабочих листов, вопросы с несколькими вариантами ответов и вопросы с короткими ответами.Я продаю товары для 4 разных курсов: химия 12 класс, химия 11 класс, физика 11 класс и наука 10 класс.

    Последнее обновление

    8 июля 2021 г.

    Поделиться

    Это викторина и тестовый пакет для модуля динамики для 11 класса физики. Включает в себя 2 викторины и 1 тест на динамику. Оценка включает решение для веса, трения, нормальной силы, чистой силы, ускорения и натяжения. Он также включает теорию о силах и законы Ньютона 3 закона.Пакет содержит вопросы с несколькими вариантами ответов, вопросы с короткими ответами и занимает 4 страницы.

    Меня зовут Даррин Мэтьюсон, я доктор органической химии. Я преподаю физику и химию более 15 лет. Все публикуемые мной Power Points, рабочие листы, викторины и тесты правильно отформатированы и готовы к печати. Проверены на ошибки и опечатки!

    У меня есть более 140 очков Power Point для продажи в моем магазине, 40 пакетов рабочих листов, 100 тестовых пакетов, 100 пакетов с множественным выбором и 70 пакетов с короткими ответами.У меня есть Power Points, тесты, викторины, вопросы с несколькими вариантами ответов, вопросы с короткими ответами и рабочие листы по каждой теме, охваченной естественными науками 10 класса, химией 11 класса, физикой 11 класса и химией 12 класса!

    Чтобы оценить мою работу, ознакомьтесь с моими 7 БЕСПЛАТНЫМИ ПРОДУКТАМИ, включая Power Points, викторины и тесты!
    https://www.tes.com/teaching-resource/resource-12545047 (тест по органической химии)
    https://www.tes.com/teaching-resource/resource-12545057 (физический тест равномерного движения и векторов)
    https: // www.tes.com/teaching-resource/resource-12545034 (тест по именованию соединений)
    https://www.tes.com/teaching-resource/resource-12545028 (тест по стехиометрии)
    https://www.tes.com/teaching -resource / resource-12544990 (обозначение алканов Power Point)
    https://www.tes.com/teaching-resource/resource-12544998 (ограничение стехиометрии реагентов Power Point)
    https://www.tes.com/teaching- resource / resource-12545007 (функция клеток и использование микроскопа Power Point)

    Платная лицензия Tes Как я могу использовать это повторно?

    Отзывы

    Выберите общий рейтинг

    (нет рейтинга)

    Ваша оценка необходима, чтобы отражать ваше счастье.

    Написать отзывОтменить

    Хорошо оставлять отзыв.

    Что-то пошло не так, повторите попытку позже.

    Этот ресурс еще не проверялся.

    Чтобы обеспечить качество наших обзоров, только клиенты, которые приобрели этот ресурс, могут просматривать его.

    Сообщите об этом ресурсе, чтобы сообщить нам, если он нарушает наши условия.
    Наша служба поддержки клиентов рассмотрит ваш отчет и свяжется с вами.

    AP Physics 1 Видео с физикой

    Точек привязки для этой страницы:

    Вводные понятия для физики, основанной на алгебре
    AP Physics 1 Free Response Exam Вопросы, которые вы должны уметь решать на этом этапе учебной программы:
    1. (7:56) 2015 # 1 — Краткий ответ
    2. (4:55) 2015 # 4 — Абзац Аргумент Краткий ответ
    3. (10:30) 2016 № 3 — Количественный / качественный перевод
    AP Physics 1 Free Response Exam Вопросы, которые вы должны уметь решать на этом этапе учебной программы:
    1. (12:36) 2017 # 2 — Вопрос об экспериментальном дизайне
    2. (11:19) 2019 # 2 — Количественный / качественный перевод
    AP Physics 1 Free Response Exam Вопросы, которые вы должны уметь решать на этом этапе учебной программы:
    1. (12:44) 2015 # 3 — Количественный / качественный перевод
    2. (6:55) 2017 # 4 — Краткий ответ
    3. (11:38) 2019 # 3 — Вопрос экспериментального проектирования
    AP Physics 1 Free Response Exam Вопросы, которые вы должны уметь решать на этом этапе учебной программы:
    1. (8:30) 2016 # 2 — Experimental Design Question
    AP Physics 1 Free Response Exam Вопросы, которые вы должны уметь решать на этом этапе учебной программы:
    1. (10:10) 2016 # 1 — Краткий ответ
    2. (10:54) 2017 # 3 — Количественный / качественный перевод
    3. (10:19) 2018 # 3 — Количественный / качественный перевод
    4. (10:29) 2019 # 1 — Краткий ответ
    AP Physics 1 Free Response Exam Вопросы, которые вы должны уметь решать на этом этапе учебной программы:
    1. (11:06) 2018 # 1 — Short Answer
    AP Physics 1 Free Response Exam Вопросы, которые вы должны уметь решать на этом этапе учебной программы:
    1. (9:42) 2018 # 5 — Paragraph Argument Short Answer
    2020.12.18: Совет колледжа объявил, что волны, электростатика и электричество больше не являются частью учебной программы AP Physics 1. Таким образом, этот пункт на странице отмечает конец тем AP Physics 1. Пожалуйста, планируйте соответственно!
    AP Physics 1 Free Response Exam Вопросы, которые вы должны уметь решать на этом этапе учебной программы:
    1. (6:15) 2015 # 5 — Краткий ответ
    2. (4:08) 2016 # 5 — Абзац Аргумент Краткий ответ
    3. (6:33) 2017 # 5 — Краткий ответ
    4. (8:00) 2018 # 4 — Краткий ответ
    5. (8:08) 2019 # 5 — Краткий ответ
    AP Physics 1 Free Response Exam Вопросы, которые вы должны уметь решать на этом этапе учебной программы:
    1. (7:54) 2015 # 2 — Вопрос экспериментального дизайна
    2. (12:11) 2016 # 4 — Количественный / качественный перевод
    3. (6:28) 2017 # 1 — Краткий ответ на аргумент абзаца
    4. (11:11) 2018 # 2 — Вопрос экспериментального проектирования
    5. (4:58) 2019 # 4 — Краткий ответ на аргумент абзаца

    Все видео ниже этого пункта предназначены для учебной программы AP Physics 2 и являются видео лекций в классе.В какой-то момент я планирую переделать их как видео по «Перелистыванию физики», однако сейчас я концентрируюсь на других программах.

    Электрический заряд, электрическая сила и электрическое поле

    Все лекции и ссылка на плейлист YouTube по электрическому заряду, электрической силе и электрическому полю
    Введение в силу Кулона или электрическую силу: видео
    1. Определение коэффициента трения между пробкой и столом — Проблема силы Кулона: видео
    2. Введение в электрическое поле: видео
    3. Таблица друзей — заряд, электрическая сила и электрическое поле: видео
    4. Проблема определения электрического поля из-за двухточечных зарядов: видео

    Энергия электрического потенциала и разность электрических потенциалов


    Все лекции и ссылка на плейлист YouTube для определения электрической потенциальной энергии и разности электрических потенциалов
    1. Введение в электрическую потенциальную энергию в постоянном электрическом поле: видео
    2. Проблема определения изменения электрической потенциальной энергии в постоянном поле: видео
    3. Введение в электрическую потенциальную энергию между двумя точечными зарядами: видео
    4. Введение в разность электрических потенциалов в постоянном электрическом поле: видео
    5. Введение в разность электрических потенциалов, обусловленную точечным зарядом: видео
    6. Проблема определения разницы электрических потенциалов из-за точечного заряда: видео
    7. Проблема определения разницы электрического потенциала из-за двухточечных зарядов: видео
    8. Проблема определения изменения электрической потенциальной энергии в постоянном электрическом поле: видео
    9. Таблица друзей — электрическая потенциальная энергия и разность электрических потенциалов: видео
    10. 9071 0
      1. Определение эквивалентного сопротивления и разности электрических потенциалов в цепи с последовательными резисторами: видео
      2. Определение эквивалентного сопротивления и электрической мощности в цепи с последовательными и параллельными резисторами: видео
      3. Цепь сложного резистора: Часть a) Рисование Принципиальная схема и поиск эквивалентного сопротивления: видео
      4. Схема сложного резистора: часть б) Определение мощности, рассеиваемой на одном резисторе: видео

      Магнитные поля и магнитные силы


      Все лекции и ссылка на плейлист YouTube для магнитных полей и магнитных сил
      1. Введение в магнитные полюса и закон полюсов: видео
      2. Введение в магнитное поле и магнитные полюса Земли: видео
      3. Введение в магнитное поле Сила и Теслас: видео
      4. Введение в правило правой руки для направления магнитной силы с примерами: видео
      5. Примеры правил правой руки с магнитной силой с использованием основных направлений (север, юг, восток, запад): видео
      6. Введение в Магнитная сила на проводе, проводящем ток: Видео
      7. Введение в путь заряженной частицы в постоянном магнитном поле: Видео
      8. Пример — Электрон, движущийся в постоянном магнитном поле: Часть a) Определение скорости: Видео
      9. Пример — Электрон, движущийся в постоянном магнитном поле: Часть б) Определение периода: Видео
      10. Пример — Электрон, движущийся в постоянном магнитном поле d: Часть c) Определение разницы в электрическом потенциале: видео

      Зеркала, отражение и диаграммы лучей


      Все лекции и ссылка на плейлист YouTube для зеркал, отражений и диаграмм лучей
      1. Введение в свет, видимый свет, электромагнитные волны и скорость света: видео
      2. Введение в отражение света — зеркальное и диффузное: видео
      3. Обзор электромагнитного спектра: видео
      4. Зеркальное отражение от плоского зеркала — падающие и отраженные лучи: видео
      5. Основы отражения от вогнутого сферического зеркала: видео
      6. Введение в характеристики расстояния до объекта и объекта, увеличения и изображения: Видео
      7. (часть a) Пример — обучение рисованию лучевой диаграммы вогнутого сферического зеркала — только диаграмма: видео
      8. (часть b) Пример — обучение рисованию лучевой диаграммы вогнутого сферического зеркала — использование математики: Видео
      9. Определение характеристик изображения в вогнутом сферическом зеркале с помощью видео: видео
      10. Определение характеристик изображения в Выпуклое сферическое зеркало с помощью видео: видео
      11. (часть a) Пример — обучение рисованию лучевой диаграммы выпуклого сферического зеркала — использование математики: видео
      12. (часть b) Пример — обучение рисованию лучевой диаграммы Выпуклое сферическое зеркало — диаграмма лучей: видео
      13. Таблица друзей — зеркала: видео
      14. Обзор основ зеркал, характеристик изображений и лучевых диаграмм: видео
      15. I Am A Mirror — Песня о физике зеркал — исполняется вживую in Class !: Тексты песен с аккордами, mp3 и видео
      1. Введение в преломление света и показатель преломления: видео
      2. Введение в дисперсию, закон Снеллиуса с примером задачи: видео
      3. Понимание того, как работает мираж — преломление света: видео
      4. Введение в сходящиеся линзы (или Bi-Convex), линзы Френеля и характеристики изображения: видео
      5. Пример — обучение рисованию лучевой диаграммы сходящейся линзы (или Bi-Convex) включает в себя Math: Video
      6. Example — Learning how to Draw a Ray Diagram of Расходящаяся линза (или двояковогнутая) включает в себя математику: видео
      7. Введение в преломление света и показатель преломления: видео
      8. Основное объяснение оптики человеческого глаза: видео
      9. Введение в полное внутреннее отражение и критический угол: видео
      10. Пример — полное внутреннее отражение, критический угол и построение лучевой диаграммы: видео
      11. Добавление в список друзей — Объективы: видео

      Интерференция и дифракция света
      Все лекции и ссылка на плейлист YouTube по интерференции и дифракции света
      1. Введение в конструктивное и деструктивное вмешательство света и дифракцию: видео
      2. Введение в эксперимент Томаса Юнга 1801 с двойной щелью: видео
      3. Выведение уравнений максимума интерференции и минимумы в эксперименте с двойной щелью: видео
      4. Проблема — определение угла для максимума интерференции в эксперименте с двойной щелью: видео
      5. Задача — определение угла для минимума интерференции в эксперименте с двойной щелью: видео
      6. Введение к дифракционной решетке и максимумам и минимумам интерференции: видео
      7. # 1 Проблема — определение количества линий на сантиметр на дифракционной решетке: видео

      Обзор всего со времени последнего обзора (Электричество через оптику)

      Все лекции и ссылка на плейлист YouTube для этого обзора. Магнитная сила: видео
    11. (часть 4) Свет, оптика, зеркала, линзы, помехи: видео
    1. Введение в лекцию по современной физике: видео
    2. Что такое свет? Объяснение эксперимента Майкельсона Морли: поиск эфира: видео
    3. Введение в относительность одновременности: видео
    4. Доказательство замедления времени — часть специальной теории относительности Эйнштейна: видео
    5. Пример двойного парадокса — замедление времени Задача: видео
    6. Обсуждение последствий специальной теории относительности: видео
    7. Обсуждение черных дыр, как их найти и спагеттификация: видео
    8. Обсуждение нашей Галактики, Вселенной и того, насколько мы незначительны: Видео

    Физика звука и музыки
    Все лекции и ссылка на плейлист YouTube по физике звука и музыки
    1. Определение частот, создаваемых велосипедной шиной с помощью эффекта Доплера: видео
    2. Введение в интенсивность звука и человеческий слух: видео
    3. Введение в сверхзвуковой Скорости и звуковой удар: видео
    4. Введение в физику музыкальной теории: видео
    5. Основы физики гитары: видео
    6. Лицо бабочки — Песня, не имеющая ничего общего с физикой, извините: видео

    Галилей: законы динамики

    Галилей: законы динамики
    Галилео и
    Законы динамики

    Галилей и падающая башня

    Галилей внес большой вклад в наше понимание законов, регулирующих движение предметов.Знаменитый эксперимент Пизанской башни может быть апокрифический. Вероятно, что сам Галилей не уронил два предмета очень разный вес от башни, чтобы доказать, что (вопреки популярным ожидания) они упадут на землю в в то же время. Однако несомненно, что Галилей понимал принцип участвовал и, вероятно, проводил аналогичные эксперименты. Осознание того, что, как мы сказал бы, говоря современным языком, ускорение свободного падения не зависит от вес объекта был важен для формулировки теории гравитации Ньютон.

    Галилей также провел эксперименты с движением снаряда, чтобы ясно показать, как ошибочными были аризотелевские концепции:

    Галилей и концепция инерции

    Возможно, величайший Вкладом в физику была его формулировка концепции инерции : объект в состоянии движения обладает « инерцией », которая заставляет его оставаться в этом состоянии движения, если на него действует внешняя сила.

    Большинство объектов в состояние движения НЕ остается в этом состоянии движения. Например, блок дерева, проталкиваемого с постоянной скоростью по столу, быстро останавливается, когда мы перестань давить. Таким образом, Аристотель считал, что покоящиеся объекты остались в покое если на них не действовала сила, но движущиеся объекты не оставались в движение, если на них постоянно не действует сила. Галилей, в силу серия экспериментов (многие с объектами, скользящими по наклонным плоскостям), понял, что анализ Аристотеля был неправильным, потому что он не учетная запись правильно для скрытой силы: сила трения между поверхностью и объект.

    Таким образом, когда мы толкаем деревянный брусок по столу, появляются две противоположные силы, которые действуют: сила, связанная с толчком, и сила, которая связано с трением и действует в противоположном направлении. Галилей понял, что по мере уменьшения сил трения (для например, поместив масло на стол) объект будет двигаться все дальше и дальше перед остановкой. Из этого он абстрагировал основную форму закон инерции: если силы трения могут быть уменьшены точно до нуля (невозможно в реалистичном экспериментируйте, но это может быть приближено с высокой точностью) объект толкается с постоянной скоростью через бесконечно протяженную поверхность без трения будет продолжать движение с этой скоростью навсегда после того, как мы прекратим толкать, если только новая сила не подействует на него позже.

    Галилей и церковь

    Галилей бросил вызов авторитету Церкви через его нападение на Аристотелевская концепция Вселенной в конечном итоге доставила ему серьезные неприятности. с инквизицией. В конце своей жизни он был вынужден публично отрекаться от своих коперниканских взглядов и провел свои последние годы, по сути, под домашним арестом. Его история, безусловно, представляет собой один из самых печальных примеров конфликта. между научным методом и «наукой», основанной на неоспоримых орган власти.К сожалению, в современном обществе все еще есть много сил, которые сковывает научный метод открытого исследования идеологическими цепи того или иного вида.

    Галилей-физик

    • Ключевой аспект понимания методологии что соотношения могут привести к физическому пониманию, если измерение сделано.

    В контексте этого класса мы будем охарактеризовать Галилео как строгий эмпирик (вы видите, что делает природа, то и природа делает).

    «Прорыв», который Галилей смог обеспечить, был в его признание того, какие аспекты движения (физика) были универсальными. В конечном итоге это привело к его идее системы координат.

    Обобщение взглядов Аристотеля

    Аристотель считал, что существует два вида движения неодушевленной материи: естественное и неестественное. Неестественное (или «насильственное») движение — это когда что-то толкают, и в этом случае скорость движения пропорциональна сила толчка.(Вероятно, это было выведено из наблюдения запряженных волов повозок и лодок.) Естественное движение — это когда что-то ищет свое естественное место во Вселенной, например, падающий камень или поднимающийся огонь. (Мы только говоря здесь о веществах, состоящих из земли, воды, воздуха и огня, «естественном круговом движении» планеты, состоящие из aither, рассматриваются отдельно).

    Что касается естественного движения тяжелых предметов, падающих на землю, Аристотель утверждал, что скорость падения пропорционально весу и обратно пропорционально плотности среды, в которой падало тело через.Он также упомянул, что было некоторое ускорение, поскольку тело приближалось к своей собственный элемент, его вес увеличился и он ускорился. Однако эти замечания у Аристотеля очень краткие и расплывчатые и, конечно, не количественные.

    Собственно, эти взгляды Аристотеля не остались незамеченными даже в древних Афинах. Тридцать лет спустя или около того После смерти Аристотеля Стратон указал, что камень, упавший с большей высоты, оказал большее влияние на земли, предполагая, что камень набрал большую скорость, когда упал с большей высоты.

    Галилей сатирически отвергает все идеи Аристотеля:

    Две новые науки

    Галилей изложил свои идеи о падающих телах и о снарядах в целом в книге под названием «Два новых Науки ». Это была наука о движении, которая стала краеугольным камнем физики, и материаловедение и строительство, важный вклад в инженерное дело.

    Идеи живо представлены в виде диалога с участием трех персонажей: Сальвиати, Сагредо и Симпличио.Официальную церковную точку зрения, то есть аристотелизм, выдвигает персонаж по имени Simplicio, а остальные обычно сносят. Защита Галилея по обвинению в ереси в аналогичной книге было то, что он просто излагал все точки зрения, но это несколько лукаво — Симпличио почти неизменно изображается простодушным.

    Сальвиати заявляет:

    Я очень сомневаюсь, что Аристотель когда-либо проверял экспериментально, верно ли, что два камня, один из которых весит десять раз как и другой, если позволить ему упасть в одно и то же мгновение с высоты, скажем, 100 локтей, он будет отличаться скорость, чтобы, когда более тяжелый достиг земли, другой не упал бы более чем на 10 локтей.

    Таким образом, это знаменует собой начало современной эры в науке — отношение, которое утверждает, что физический мир авторитетами, независимо от того, насколько они мудры или почитаемы, выдержат или упадут после экспериментального испытания.

    Однако экспериментальный тест должен иметь достаточную точность, чтобы это должен быть надежный тест.

    В качестве примера неточных измерений давайте посмотрим на гравитацию …

    Некоторые эксперименты с гравитацией:

    1. Коровы-падальщики с высоты 10 и 100 метров
    2. Запишите время свободного падения в единицах сердцебиения и введите эти значения.
    3. Используйте виртуальный секундомер и введите эти значения

    В чем здесь основная проблема по определению экспериментальных точность?

    Как и Гален, Галилей вел подробные записи своих экспериментов. чтобы другие могли его продублировать.

    Но как замедлить гравитацию?

    Виртуальные эксперименты Галилея

    Галилей был требовательным экспериментатором, полностью осознавая ценность точных измерений и повторяемых экспериментальных деталей:

    Перевод цитаты из одного из журналов Галилея:

    Кусок деревянного каркаса или бруса длиной около 12 локтей, шириной пол локтя и шириной в три пальца. толстый, был взят; на его краю прорезан канал шириной чуть больше пальца; сделав этот паз очень прямая, гладкая и отполированная, и облицованная пергаментом, такая же гладкая и отполированная, как возможно, мы катались по нему твердым, гладким и очень круглым бронзовым шаром.Поместив эту доску в наклонный положение, подняв один конец на один или два локтя над другим, мы катили мяч, как я только что сказал: вдоль канала, отмечая, как будет описано ниже, время, необходимое для спуска. Мы повторил этот эксперимент несколько раз, чтобы измерить время с такой точностью, чтобы отклонение между двумя наблюдениями никогда не превышал одной десятой доли пульса. Выполнив эту операцию и Убедившись в его надежности, мы теперь катили мяч только на четверть длины канала; а также измерив время его спуска, мы нашли ровно половину первого.Затем мы попробовали другие расстояния, сравнивают время для всей длины с временем для половины, или с временем для двух третей, или три четверти, или даже на любую фракцию; в таких экспериментах, повторенных сто раз, мы всегда находили что пройденные пространства были относительно друг друга как квадраты времен, и это было верно для всех наклонностей плоскости, т. е. канала, по которому мы катили мяч. Мы также заметили, что времена спуска для различные наклоны самолета, передавали друг другу в точности то соотношение, которое, как мы увидим позже, Автор предсказал и продемонстрировал для них.

    Для измерения времени мы использовали большой сосуд с водой, расположенный на возвышении; ко дну К этому сосуду припаивалась труба небольшого диаметра, дающая тонкую струю воды, которую мы собирали в небольшой стаканчик. во время каждого спуска, будь то на всем протяжении канала или на части его длины; вода Собранные таким образом взвешивались после каждого спуска на очень точных весах; различия и соотношения этих веса дали нам разницу и соотношение времен, и это с такой точностью, что, хотя операция повторялось много-много раз, заметных расхождений в результатах не было.

    контрольных вопросов

    Размеры и оценки
    Знакомит учащихся с идеей составления оценок, повсеместное владение физикой. Также хорошо сочетается с обсуждение размеров, обработка больших чисел и, возможно, размерный анализ.
    Размеры и смета Размеры и оценка: решения
    Смещение и скорость
    Позволяет учащимся изучить понятие средней скорости. (Не требует знания ускорения.)
    Рабочий объем и скорость Рабочий объем и скорость: решения
    Скорость и ускорение
    Концептуально проверяет понимание учащимися графического представления скорость и ускорение.
    Скорость и разгон Скорость и ускорение: решения
    Путешествие на Луну
    Студенты используют основные кинематические уравнения, чтобы вычислить время полета к Луне и Альфе Центавра.
    Путешествие на луну Путешествие на Луну: решения
    Самолеты пожарные
    Использование кинематических уравнений, студенты исследуют независимость направления движения. (Можно связать с демонстрацией класса для большей ясности.)
    Пожаротушение самолеты Пожаротушение самолеты: решения
    Гонка на двух автомобилях
    Далее практика с кинематикой.Использует онлайн-апплет Java созданный профессором физики BU Эндрю Даффи.
    А гонка на двух автомобилях А гонка на двух автомобилях: решения
    Пловец в стрессовом состоянии
    Исследование кинематики, сложения векторов и независимость направлений движения.
    находящийся в состоянии стресса пловец пловец в стрессовом состоянии: решения

    Силы и Ньютона Законы

    Трактор тягач
    Включает Второй закон Ньютона и сложение векторов.Слегка сложная проблема для студентов в их ранних встречах с Законы Ньютона.
    Трактор тянущий Трактор вытягивание: решения
    Прыжок по вертикали
    Немного более комплексная проблема, опирающаяся как на студентов знание законов Ньютона и кинематики.
    вертикальный прыжок вертикальный прыжок: решения
    Акселерометр
    Использование Второй закон Ньютона, студенты изучают, как измерить ускорение. Требуется хорошее понимание Ньютона. законы, а также синус и косинус.
    An акселерометр An акселерометр: решения

    Энергия и работа

    Рычаги
    Обучает студентов как работают рычаги.Студенты проходят через математический вывод с использованием немного геометрии и концепция работы.
    Рычаги Рычаги: решения
    Скорость эвакуации
    Использование сохранение энергии, студенты находят убегающую скорость Земли и оцените радиус черной дыры.

    Побег скорость

    Побег скорость: решения

    Импульс Консервация
    На берегу озера
    Учащиеся используют функцию сохранения импульса и простые кинематика, чтобы выяснить, как выйти из озера, когда все они имейте с собой их книгу по физике!
    Мель на озере Мель на озере: решения
    Взрыв в воздухе
    Еще одна проблема сохранения импульса, на этот раз имея дело с гранатой, разбивающейся на более мелкие части в в воздухе.Также вкратце исследует сохранение импульса. в двух измерениях.
    Взрыв в воздухе В воздухе взрыв: решения

    Всеобщий закон Плотность

    Взвешивание Солнца
    Используя универсальный закон всемирного тяготения, студенты определить массу нашего Солнца.Эта же техника также используется для определения массы других звезд в нашем галактика.
    Взвешивание Солнца Взвешивание Солнце: решения
    Гравитация на Земле
    Студенты показывают, что универсальный закон гравитации сокращается до хорошо известных мг на Земле поверхность.Они также вычисляют г на Земле, на на вершине Эвереста и на Луне.
    Гравитация на Земле Сила тяжести на Земле: решения

    Нагрев

    Mystery Substance
    Пример того, как определить простую калориметрию. неизвестные материалы.
    Таинственная субстанция Mystery Substance: решения
    Вода и климат
    Эта проблема объединяет концепции, изученные в физика в географических условиях прибрежный климат и внутренние суши климат. Студенты также оценивают наихудший глобальный сценарий потепления.
    Вода и климат Вода и климат: решения
    Электричество и схемы
    Электрические и гравитационные силы
    Студенты сравнивают электростатическое притяжение между протоном и электроном к их гравитационному притяжение и подумайте, почему гравитация доминирует во Вселенной на длинных весах.
    Электричество против гравитации Электричество против гравитации: решения
    Три заряда на линии
    Более сложная задача закона Кулона с участием трех обвинения. Студенты исследуют, как разместить третье зарядите рядом с двумя другими так, чтобы третий заряд в равновесие.
    Три заряда на линии Три заряда на линии: решения
    Резисторы
    Сложная проблема схемы, которая проверяет понимание студентами последовательного и параллельного резистора схемы. Также исследуются предельные случаи подключения бесконечно много резисторов, подключенных параллельно или последовательно.
    Резисторы Резисторы: решения
    Энергия и электричество
    Зная P = IV и закон Ома, студенты получают два другие проявления силы и применяйте их, чтобы найти силу рассеивается как в последовательной, так и в параллельной цепи.
    Власть и электричество Власть и электричество: решения
    Волны, звук и свет
    Звук в воздухе и воде
    Студенты изучают выражение для модуля объемного сжатия и узнайте, как скорость звука зависит от этого модуля и плотность среды.Затем они находят скорость звук в воде, и используйте его для решения простой проблемы с сонаром.
    Звук в воздухе и воде Звук в воздухе и воде: решения
    Amazing Bats
    Эта проблема связана с тем, как летучие мыши используют звук для перемещаться.Он исследует как можно использовать сонар для определения дальности до объекта и имеет более сложная часть об эффекте Доплера.
    Удивительный Летучие мыши Удивительный Летучие мыши: решения
    Световые помехи
    Введение в спектроскопию.Студенты исследуйте, как свет преломляется на решетке, используйте известные длина волны света, чтобы охарактеризовать их решетку, и использовать эта решетка для определения длины волны неизвестного света источник.
    Вмешательство света Вмешательство света: решения

    Произошла ошибка при настройке вашего пользовательского файла cookie

    Произошла ошибка при настройке вашего пользовательского файла cookie

    Этот сайт использует файлы cookie для повышения производительности.Если ваш браузер не принимает файлы cookie, вы не можете просматривать этот сайт.

    Настройка вашего браузера для приема файлов cookie

    Существует множество причин, по которым cookie не может быть установлен правильно. Ниже приведены наиболее частые причины:

    • В вашем браузере отключены файлы cookie. Вам необходимо сбросить настройки вашего браузера, чтобы он принимал файлы cookie, или чтобы спросить вас, хотите ли вы принимать файлы cookie.
    • Ваш браузер спрашивает вас, хотите ли вы принимать файлы cookie, и вы отказались.Чтобы принять файлы cookie с этого сайта, нажмите кнопку «Назад» и примите файлы cookie.
    • Ваш браузер не поддерживает файлы cookie. Если вы подозреваете это, попробуйте другой браузер.
    • Дата на вашем компьютере в прошлом. Если часы вашего компьютера показывают дату до 1 января 1970 г., браузер автоматически забудет файл cookie. Чтобы исправить это, установите правильное время и дату на своем компьютере.
    • Вы установили приложение, которое отслеживает или блокирует установку файлов cookie.Вы должны отключить приложение при входе в систему или проконсультироваться с системным администратором.

    Почему этому сайту требуются файлы cookie?

    Этот сайт использует файлы cookie для повышения производительности, запоминая, что вы вошли в систему, когда переходите со страницы на страницу. Чтобы предоставить доступ без файлов cookie потребует, чтобы сайт создавал новый сеанс для каждой посещаемой страницы, что замедляет работу системы до неприемлемого уровня.

    Что сохраняется в файлах cookie?

    Этот сайт не хранит ничего, кроме автоматически сгенерированного идентификатора сеанса в cookie; никакая другая информация не фиксируется.

    Как правило, в файле cookie может храниться только информация, которую вы предоставляете, или выбор, который вы делаете при посещении веб-сайта.

    Author: alexxlab

    Добавить комментарий

    Ваш адрес email не будет опубликован. Обязательные поля помечены *